87
NOTAS DE AULA RESOLUÇÃO DE PROBLEMAS Benedito Tadeu Vasconcelos Freire

NOTAS DE AULA - UFRGS

  • Upload
    others

  • View
    5

  • Download
    0

Embed Size (px)

Citation preview

Page 1: NOTAS DE AULA - UFRGS

NOTAS DE AULARESOLUÇÃO DE PROBLEMAS

Benedito Tadeu Vasconcelos Freire

Page 2: NOTAS DE AULA - UFRGS

NOTAS DE AULAS - SEDIS. UFRN

WWW.SEDIS.UFRN.BR

Notas de Aulas, materila complementar para os alunos do Curso de Licenciatura em Matemática à Distância,da Universidade Federal do Rio Grande do Norte.

Primeira Versão, janeiro de 2016

Page 3: NOTAS DE AULA - UFRGS

Conteúdo

1 Introdução . . . . . . . . . . . . . . . . . . . . . . . . . . . . . . . . . . . . . . . . . . . . . . . . . . . . 5

2 ESTRATÉGIAS . . . . . . . . . . . . . . . . . . . . . . . . . . . . . . . . . . . . . . . . . . . . . . . . . . 9

2.1 Entender, antes de fazer! 92.2 A busca do plano 212.3 Procure semelhanças com outros problemas 33

2.4 Começar pelo fácil torna fácil o difícil 372.5 Experimente, procure algo que seja invariante 40

2.6 Faça um desenho e, dependendo da situação, pinte às cores. 43

2.7 Modifique o enunciado, para ver se lhe ocorre um caminho possível 502.8 Explore a simetria 532.9 Redução ao Absurdo. 56

2.10 Suponha o problema resolvido (ou olhe de trás para diante) 60

2.11 O Princípio da Indução. 62

3 Problemas Diversos . . . . . . . . . . . . . . . . . . . . . . . . . . . . . . . . . . . . . . . . . . . 67

4 ABREVIATURAS . . . . . . . . . . . . . . . . . . . . . . . . . . . . . . . . . . . . . . . . . . . . . . . . 85

4.1 Abreviaturas 85

5 Bibliografia . . . . . . . . . . . . . . . . . . . . . . . . . . . . . . . . . . . . . . . . . . . . . . . . . . . 87

Page 4: NOTAS DE AULA - UFRGS
Page 5: NOTAS DE AULA - UFRGS

1. Introdução

Escrevemos estas Notas de Aula com o intuito de que possam ser utilizadas pelos estudantes doCurso de Licenciatura em Matemática à Distância, da Universidade Federal do Rio Grande do Norte, comotexto complementar para o uso na disciplina Resolução de Problemas.

Na formação do professor de matemática deve prevalecer o ponto de vista de que, na sua prática docentefutura, ele exerça um ensino que priorize a aprendizagem por compreensão. Logo, ele tem de ter ao longode seu curso um treinamento que o leve as atitudes positivas, facilitando o desenvolvimento do gosto e doprazer das descobertas. Entendemos que a disciplina Resolução de Problemas reúne as condições para estetreinamento, criando um ambiente que torne o aluno um ativo participante do processo de aprendizagem.

Da prática da nossa convivência com os estudantes, verificamos que a maioria deles não apresenta desen-voltura na prática da resolução de problemas. Estas notas pretendem mostrar algumas ideias que poderãoapontar ao estudante caminhos facilitadores no sentido de tornar a atividade de resolução de problemasmenos árida.

Para resolver problemas de forma clara, o estudante tem que comunicar suas ideias de maneira coerente,com organização e com argumentos estruturados, permitindo a ele próprio e aos outros o entendimentofácil de seus argumentos e a interpretação perfeita de suas ideias. A Matemática é a linguagem apropriadapara a resolução de problemas nas diversas áreas do conhecimento, por ser concisa, ter muitos recursos enão permitir ambiguidades. Por isso, é uma linguagem da Ciência. Como um reconhecimento por suasqualidades indispensáveis à formação do cidadão, a Matemática é a única disciplina que é estudada em todosos países do mundo e em todos os níveis educacionais.

Galileu Galilei 1 foi quem introduziu a matemática como linguagem da ciência.

1GALILEU GALILEI Galileu Galilei (1564 -1642 ), italiano, foi um físico, matemático, astrônomo e filósofo. Galileu Galilei foipersonalidade fundamental na revolução científica. Foi o mais velho dos sete filhos do músico italiano, alaudista, Vincenzo Galilei e deGiulia Ammannati. Viveu a maior parte de sua vida em Pisa e em Florença, na época integrantes do Grão-Ducado da Toscana.Galileu Galilei desenvolveu os primeiros estudos sistemáticos do movimento uniformemente acelerado e do movimento do pêndulo.Descobriu a lei dos corpos, enunciou o princípio da inércia e o conceito de referencial inercial, ideias precursoras da mecânicanewtoniana. Galileu melhorou significativamente o telescópio refrator e com ele descobriu as manchas solares, as montanhas da Lua,as fases de Vénus, quatro dos satélites de Júpiter, os anéis de Saturno, as estrelas da Via Láctea. Estas descobertas contribuíramdecisivamente na defesa do heliocentrismo. Contudo a principal contribuição de Galileu foi para o método científico, pois a ciênciaassentava numa metodologia aristotélica.O físico desenvolveu ainda vários instrumentos como a balança hidrostática, um tipo de compasso geométrico que permitia medir

Page 6: NOTAS DE AULA - UFRGS

6 Capítulo 1. Introdução

De acordo com Paul Halmos 2, o coração da matemática são seus próprios problemas. "A maior parte decada vida significativa é passada com a solução de problemas; uma parte considerável da vida profissional detécnicos, engenheiros, cientistas, etc., é vivida na busca de solução de problemas de matemática. É dever detodos os professores e dos professores de matemática em particular, expor seus alunos para problemas muitomais do que aos fatos", disse ele.

O uso da Matemática como linguagem exige um conhecimento mínimo para poder ser utilizada. Porisso, o estudante necessita de situações que permitam exercitar essa linguagem, e uma delas são os empregosdos métodos de resolução de problema.

Uma referência no estudo da arte de resolver problema são os trabalhos do professor Polya. 3

É comum uma diferenciação entre problema e exercício, veja, por exemplo [ZEITZ], página 1. Oexercício é uma questão que testa o domínio do estudante numa técnica que está sendo focada ou que foirecentemente coberta. Exercícios podem ser difíceis ou fáceis, mas, de uma maneira geral, eles nunca sãointrigantes, e normalmente fica claro como proceder no sentido de como encontrar a solução. Por outro lado,um problema é uma questão que não pode ser respondida imediatamente. Problemas são muitas vezesabertos, paradoxais, às vezes indecifráveis e exigem investigação antes que se pode chegar a uma solução.

Para oportunizar a resolução de problemas, apresentamos no texto problemas de Olimpíada de Matemá-tica de vários países, por serem intrigantes, criativos e desafiadores. No texto, identificamos a origem daquase totalidade dos problemas aqui apresentados. Agradecemos a todos aqueles que queiram nos apresentara origem dos problemas ainda não identificados, para que possam constar numa próxima edição destas Notasde Aula.

ângulos e áreas, o termómetro de Galileu e o precursor do relógio de pêndulo. O método empírico, defendido por Galileu, constituium corte com o método aristotélico mais abstrato utilizado nessa época, devido a isto Galileu é considerado como o "pai da ciênciamoderna". Fonte: htt p : //pt.wikipedia.org/wiki/GalileuGalilei. Acessado em 17/03/2014.

2PAUL HALMOS - (1916 - 2006) foi um matemático estadunidense nascido na Hungria, que fez avanços fundamentais nas áreasde Lógica Matemática, Teoria da Probabilidade, Estatística, Teoria dos Operadores, Teoria Ergódica e Análise Funcional (em especial,nos espaços de Hilbert ). Ele também foi reconhecido como um grande expositor da matemática.Halmos chegou em os EUA aos 13 anos de idade, mas nunca perdeu seu sotaque húngaro. Halmos obteve seu Bacharelado naUniversidade de Illinois, graduando em filosofia e especializando-se em matemática. Ele levou apenas três anos para obter o grau, etinha apenas 19 anos quando se formou. Começou um doutorado (Ph.D.) na filosofia, mas, passou para a matemática, graduando-se em1938 sob a orientação do Professor Joseph L. Doob, com a dissertação intitulada: A Invariantes de Certas Transformações Estocásticos:A Teoria Matemática de Sistemas de Jogos.Pouco depois de sua formatura, Halmos foi para o Instituto de Estudos Avançados. Seis meses depois, ele estava trabalhando com Johnvon Neumann, que se revelou uma experiência decisiva. Na sua permanência no Instituto de Estudos Avançados, Halmos escreveu seuprimeiro livro, Espaços Vetoriais de Dimensões Infinitas, que imediatamente estabeleceu sua reputação como um excelente expositor damatemática.Halmos ensinou na Universidade de Syracuse , na Universidade de Chicago (1946-1960), na Universidade de Michigan, na Universidadeda Califórnia, em Santa Barbara (1976-1978), na Universidade do Havaí, e na Universidade de Indiana, onde se aposentou. Desdesua aposentadoria, em 1985, até sua morte, ele era ligado ao departamento de Matemática da Universidade de Santa Clara. Fonte:htt p : //en.wikipedia.org/wiki/PaulH almos. Acessado em 17/03/2014

3GEORG POLYA - George Pólya (1887 —1985) húngaro, professor de matemática de 1914 a 1940 no Instituto Federal deTecnologia de Zurique, na Suíça, de 1940 a 1953 na Stanford University, nos Estados Unidos. Pólya permaneceu como ProfessorEmérito da Stanford o resto de sua vida e carreira. Ele trabalhou em uma variedade de tópicos matemáticos, incluindo séries, teoria dosnúmeros, análise matemática, geometria, álgebra, combinatória e probabilidade.No início de sua carreira, Pólya escreveu com Gábor Szegó, matemático húngaro famoso, dois influentes livros de problemas: Problemase Teoremas em Análise (I: Série, Cálculo Integral, Teoria das Funções e II, Teoria das Funções, Zeros de polinômios, Determinantes,Teoria dos Números ,Geometria.....). Mais tarde, ele se dedicou ao estudo da Heurística, um método ou processo criado com o objetivode encontrar soluções para um problema, para identificar métodos sistemáticos de resolução de problemas a uma maior descoberta einvenção em matemática para os estudantes, professores e pesquisadores. Ele escreveu cinco livros sobre o assunto: How to Solve it,traduzido para o português como A Arte de Resolver Problema, Matemática e Raciocínio Plausível (Volume I: Indução e Analogia emMatemática, e Volume II: Padrões de Plausível Inference) e descoberta matemática: Na compreensão, aprendizado e ensino ProblemSolving (volumes 1 e 2).Em How to Solve It, Pólya fornece sugestões heurísticas para resolver uma gama de problemas, incluindo os problemas matemáticos eos não-matemáticos. O livro inclui conselhos para ensinar os alunos de matemática e uma mini-enciclopédia de termos heurísticos. Foitraduzido para várias línguas e já vendeu mais de um milhão de cópias. O físico russo Zhores I. Alfyorov, (Prêmio Nobel em 2000),elogiou, lembrando que ele era um fã. O livro ainda é usado em educação matemática.Além de suas obras que abordam diretamente a resolução de problemas, Pólya escreveu outro livro chamado Métodos Matemáticos emCiências, com base em um trabalho de 1963 apoiado pela National Science Foundation, editado por Leon Bowden, e publicado pelaAssociação Matemática da América (MAA), em 1977. FONTE: htt p : //en.wikipedia.org/wiki/GeorgePolya.

Page 7: NOTAS DE AULA - UFRGS

7

Esperamos que os estudantes possam apreciar as ideias que permitem a resolução dos problemas aquiapresentados.

Todos os erros e equívocos são de nossa responsabilidade. Serão bem vindos comentários apontandoeventuais erros, como também alternativas para melhorar o texto.

Natal, janeiro de 2016

Benedito Tadeu Vasconcelos [email protected]

Page 8: NOTAS DE AULA - UFRGS
Page 9: NOTAS DE AULA - UFRGS

2. ESTRATÉGIAS

Já é clássica e bem conhecida, a formulação que fez Polya das quatro etapas essenciais para a resolução deum problema, que constituem o ponto de partida de todos os estudos posteriores:• Entender, antes de fazer• Traçar um plano para resolvê-lo• Colocar o plano em prática• Comprovar os resultados

No que se segue, vamos desenvolver estas etapas, exercitando a resolução de problemas.

2.1 Entender, antes de fazer!Quando alguém lhe propõe um problema, um jogo, um quebra-cabeça, inicialmente, assegure-se que enten-deu a fundo os dados do problema, as regras do jogo e o possível lugar que tem cada uma dessas informações,como elas se encaixam umas com as outras.

Resumindo: para resolver um problema, é imprescindível que você conheça o que é dado e, exatamente,o que o problema pede.

Diante de um problema, faça a si mesmo as seguintes perguntas:• Qual é a incógnita?• Quais são as quantidades dadas?• Quais são as condições dadas?• O que o problema pede?

A solução de um problema consiste em ligar, por passos lógicos, os dados do problema ao que nele sepede.

Exemplo-1:

Duas velas de comprimentos iguais e de distintas espessuras começam a queimar ao mesmo tempo.Uma delas vai queimar completamente em 4 horas, a outra em 5 horas. Durante quantas horas asvelas queimarão até que o comprimento de uma delas seja 3 vezes o comprimento da outra?

Solução

Page 10: NOTAS DE AULA - UFRGS

10 Capítulo 2. ESTRATÉGIAS

Depois de ler o problema (se preciso for, mais de uma vez, talvez até em voz alta), de forma a saberexatamente quais as quantidades dadas, quais as condições dadas (hipóteses) e o que se pede, vamos traçarum plano para a solução.

Vamos chamar de d o comprimento das duas velas, de A a vela que queima completamente em 4 horas ede B a vela que queima completamente em 5 horas.Assim, a razão pela qual elas queimam é dada por:

Vela A :d4

e Vela B :d5.

Depois de t horas queimando, o comprimento C de cada vela será dado por:

CA = d(

1− t4

)CB = d

(1− t

5

).

Queremos saber o tempo no qual o comprimento da vela B será 3 o comprimento da vela A. Assim,temos que:

d(

1− t5

)= 3×d

(1− t

4

)⇐⇒

(1− t

5

)= 3×

(1− t

4

)⇐⇒ 5− t

5= 3 ·

(4− t4

)⇐⇒

⇐⇒ 4(5− t) = 15(4− t) ⇐⇒ 20−4t = 60−15t ⇐⇒ t =4011

.

Portanto, as velas queimarão durante 4011 horas para que o comprimento de uma delas seja 3 vezes o

comprimento da outra.

Exemplo-2:

Dois barcos partem ao mesmo tempo das margens opostas de um rio e viajam com velocidadesconstantes, mas diferentes. Eles passam um pelo outro quando estão a 700 metros de uma dasmargens e cada um continua para a outra margem do rio, de onde eles retornam. Na sua viagem deregresso os barcos passa novamente um pelo outro — desta vez a uma distância de 400 metros damargem oposta. Qual é largura do rio?

Solução

(Problema proposto por W.C. Rufus, na revista American Mathematical Monthly, 47, fevereiro de 1947, veja[BRIGGS], pag. 9.

Depois de ler o problema (se preciso for, mais de uma vez, talvez até em voz alta), de forma a saberexatamente quais as quantidades dadas, quais as condições dadas (hipóteses) e o que se pede, vamos traçarum plano para a solução.Aqui, facilita se fizermos um desenho ilustrativo da viajem dos dois barcos com o movimento de ida e vindaatravés do rio.

barco 2

barco 1 •• • •

• • •

••

700 m400 m

Page 11: NOTAS DE AULA - UFRGS

2.1 Entender, antes de fazer! 11

Para a solução do problema, um fato importante é o que afirma que as velocidades dos barcos são constantesmas diferentes.Quando trabalhamos com velocidade constante a relação que estabelece a distância percorrida é

distância = (velocidade)× (tempo gasto)

Agora, com o desenho acima em mente, devemos atentar para um fato que é a chave da solução: o tempogasto por cada um dos barcos para acontecer o primeiro encontro entre eles é o mesmo que o tempo gastopelo outro. Além disso, é dado a localização deste encontro: 700 metros de uma das margens.Observe que o tempo gasto pelos dois barcos para acontecer o segundo encontro, a 400 metros da margemoposta, também é o mesmo para os dois barcos.Assim, com estas duas observações, podemos escrever as duas relações entre as variáveis do problema: d alargura do rio, que é o que queremos encontrar, v1, e v2 as velocidades dos barcos 1 e 2, respectivamente.O barco 1, viajando com velocidade v1, no primeiro encontro, percorreu d−700 metros, enquanto obarco 2, viajando com velocidade v2, no primeiro encontro percorreu 700 metros. Logo, para o primeiroencontro, teremos para o barco 1:

tempo gasto =d−700

v1=

700v2

. (∗)

Para o barco 2, temos:

tempo gasto =700+d−400

v1=

d−700+400v2

⇐⇒ tempo gasto =300+d

v1=

d−300v2

. (∗∗)

Podemos rescrever a equação (∗) como:

v1

v2=

d−700700

. (∗∗∗)

De maneira análoga, a equação (∗∗) pode ser reescrita como:

v1

v2=

300+dd−300

. (∗∗∗∗)

Fazendo (∗∗∗) = (∗∗∗∗), obtemos

d−700700

=300+dd−300

⇐⇒ d2−1700d = 0 ⇐⇒ d · (d−1700) = 0 ⇒ d = 1700, pois d 6= 0.

Portanto, a largura do rio é 1700 metros.

Exemplo-3:

No meu carro, uma determinada marca de pneu dura 40.000 km em uma roda dianteira ou 60.000km em uma roda traseira. Ao longo da viagem, trocando os pneus dianteiros pelos traseiros, qual é amaior distância que consigo percorrer com um conjunto de quatro destes pneus?

Solução

Depois de ler o problema (se preciso for, mais de uma vez, talvez até em voz alta), de forma a saberexatamente quais as quantidades dadas, quais as condições dadas (hipóteses) e o que se pede, vamos traçarum plano para a solução.

A chave da solução é conhecer o desgaste de cada pneu dianteiro e traseiro em cada 1 quilômetro rodado.Como cada pneus dura 40.000 km em uma roda dianteira, segue que o desgaste por cada 1 quilômetrorodado é igual a

140.000

Page 12: NOTAS DE AULA - UFRGS

12 Capítulo 2. ESTRATÉGIAS

Como cada pneus dura 60.000 km em uma roda traseira, segue que o desgaste por cada 1 quilômetrorodado é igual a

160.000

Logo, a média de desgaste dos 4 pneus é dada por

14

( 140.000

+1

40.000+

160.000

+1

60.000

)=

14

( 240.000

+2

60.000

)=

=14

( 120.000

+1

30.000

)=

14

( 3+260.000

)=

148.000

.

Portanto, considerando que eu otimizo o uso dos pneus, a distância máxima que consigo deles é rodar 48.000quilômetros. Isso será alcançado através do rodizio dos pneus dianteiros com os traseiros após 24.000 km.

Exemplo-4:

Uma empresa vende mensalmente 400 unidades de um produto a R$ 120,00 cada, proporcionandoum faturamento bruto de R$ 48.000,00. Observou-se que para cada real descontado no preço eramvendidos 20 unidades a mais por mês. Calcule o desconto que proporciona o maior faturamentopossível e qual é esse faturamento.

Solução

Depois de ler o problema (se preciso for, mais de uma vez, talvez até em voz alta), de forma a saberexatamente quais as quantidades dadas, quais as condições dadas (hipóteses) e o que se pede, vamos traçarum plano para a solução.

Observe que o faturamento varia como o desconto dado ao produto. Vamos chamar de x o desconto quemaximiza o faturamento, que vai ser a incógnita que queremos encontrar.O preço do produto com o desconto vai ser 120− x.Para cada unidade do desconto são vendidos 20 produtos a mais, além dos 400 que já eram comercializados.Logo, serão vendidos 400+20x produtos.Portanto, o faturamento pode ser expresso pela função:

F(x) = (400+20x) · (120− x) =−20x2 +2000x+48000

Agora, observe que a expressão F(x) está definida para todo número real x. No entanto, os valores de xadequados ao problema são aqueles satisfazendo 0≤ x < 120, caso contrário, teríamos ou um preço doproduto maior do que os R$ 120,00, ou teríamos um preço negativo. A expressão de F(x) pode ser reescritacomo:

F(x) =−20x2 +2000x+48000 =−20(x2−100x−2400) =−20(x2−100x+2500−2500−2400) =

=−20[(x−50)2]+20×4900 =−20(x−50)2 +98000.

Agora, basta observar que o valor de F(x) atinge um máximo quando x−50 = 0. Ou seja, F(x) atingeum máximo quando x = 50.Portanto, o desconto que proporciona o maior faturamento possível é R$ 50,00 e o faturamento máximo éigual a F(50) =−20(50−50)2 +98000 = 98.000 reais.

Exemplo-5:

Um barril contém 64 litros de vinho. Substituímos 16 litros de vinho por 16 litros de água.Suponhamos que o vinho e a água se misturam uniformemente e que o volume da mistura seja asoma dos dois volumes. Em seguida, substituímos 16 litros da mistura por 16 litros de água.Esperamos que se misturem e voltamos a fazer mais uma vez mais esse procedimento. Quantoslitros de vinho (misturados com a água) permanecem no barril?

Solução

Page 13: NOTAS DE AULA - UFRGS

2.1 Entender, antes de fazer! 13

(Canguru Matemático 2005) Depois de ler o problema (se preciso for, mais de uma vez, talvez até em vozalta), de forma a saber exatamente quais as quantidades dadas, quais as condições dadas (hipóteses) e o quese pede, vamos traçar um plano para a solução.

No total, fizemos três operações de substituir 16 litros por 16 litros de água. Vejamos, com a ajuda databela a seguir, quantos litros de vinho permanece no barril logo após a primeira operação.

Água Vinho TotalInicio 0 64 64

Retira-se 16 litros de Vinho - Operação 1 -16 -16Acrescenta-se 16 litros de Água +16 +16

Resultado Final da Primeira Operação 16 48 64

É fácil ver que, logo após a primeira operação, a proporção entre a quantidade de água e a quantidade devinho é de 1 para 3. Portanto, esta proporção se mantém quando se retira 16 litro da mistura para fazer asubstituição por 16 litros de água. Isto significa dizer que, ao se retirar 16 litros da mistura, retiramos 4litros de água e 12 litros de vinho.Na tabela a seguir veremos quantos litros de vinho permanece no barril logo aós a segunda operação:

Água Vinho TotalO que resta após a primeira operação 16 48 64

Retirando-se 16 litros da mistura - Operação 2 -4 -12 -16Acrescenta-se 16 litros de Água +16 +16

Resultado Final Após a Segunda Operação 28 36 64

Assim, de maneira análoga, é fácil ver que, após a segunda operação, a proprção entre água e vinho é de7 para 9. Portanto, quando retiramos 16 litros do barril para efetuarmos a terceira operação, retira-se 7litros de água e 9 litros de vinho. Finalmente, na tabela a seguir veremos quantos litros de vinho permaneceno barril logo após a terceira operação:

Água Vinho TotalO que resta após a segunda operação 28 36 64

Retirando-se 16 litros da mistura - Operação 3 -7 -9 -16Acrescenta-se 16 litros de Água +16 +16

Resultado Final Após a Segunda Operação 37 27 64

Portanto, permanece no barril 27 litros de vinho.

Exemplo-6:

Necessita-se programar uma dieta com dois alimentos S e T . Cada unidade do alimento S contém100 calorias e 15 gramas de proteínas. A unidade do alimento T contém 200 calorias e 15gramas de proteínas. A dieta requer um mínimo de 1.000 calorias e 90 gramas de proteínas. Se opreço de cada unidade do alimento S é 400 reais e de cada unidade do alimento T é de 300reais, quantas unidades de cada alimento deve conter a dieta para minimizar o custo?

Solução

(Olimpíada de Matemática do Perú) Depois de ler o problema (se preciso for, mais de uma vez, talvez até emvoz alta), de forma a saber exatamente quais as quantidades dadas, quais as condições dadas (hipóteses) e oque se pede, vamos traçar um plano para a solução.

Suponha que a dieta contém s unidades do alimento S e t unidades do alimento T . Assim, a quantidadede calorias da dieta seria igual a

100s+200t,

que, pelas hipóteses do problema essa quantidade deve ser no mínimo 1.000. Ou seja,

100s+200t ≥ 1000 ⇐⇒ s+2t ≥ 10. (1)

Page 14: NOTAS DE AULA - UFRGS

14 Capítulo 2. ESTRATÉGIAS

A quantidade de proteína da dieta seria igual a

15s+10t,

que, pelas hipóteses do problema essa quantidade deve ser no mínimo 90. Ou seja,

15s+10t ≥ 90 ⇐⇒ 3s+2t ≥ 18. (2)

Por outro lado, o custo da dieta é dado por

400s+300t (3).

Assim, queremos minimizar a expressão (3) usando as restrições (1) e (2), tendo sempre em conta ques≥ 0 e t ≥ 0, pois se tratam de quantidades.A idéia da solução é multiplicar (1) por uma certa quantidade M e a equação (2) por um certo número Npara obter a expressão 400s+300t.Fazendo isso, obtemos as equações:

M+3N = 400 e 2M+2N = 300,

que admite como solução M = 25 e N = 125. Logo, multiplicamos a equação (1) por 25 e a equação(2) por 125, e somamos:

400s+300t ≥ 250+2250 = 2500,

e para garantir que o valor mínimo de 400s+300t seja 2500, devemos ter:

s+2t = 10 e 3s+2t = 18,

que admite como solução s = 4 e t = 3.Portanto, o menor custo da dieta se consegue com s = 4 e t = 3.

Exemplo-7:

Uma caixa contém p bolas brancas e q bolas pretas. Além dessas bolas, existe uma pilha debolas pretas. Retiram-se duas bolas da caixa. Se elas são da mesma cor, uma bola preta da pilha écolocada na caixa. Se elas são de cores distintas, a bola branca é colocada de volta na caixa. Esteprocedimento é repetido até que o último par de bolas seja removido e que uma última bola reste nacaixa. Qual é a probabilidade de que esta última bola seja branca ?

Solução

(Olimpíada de Matemática da Austrália-1983) Depois de ler o problema (se preciso for, mais de uma vez,talvez até em voz alta), de forma a saber exatamente quais as quantidades dadas, quais as condições dadas(hipóteses) e o que se pede, vamos traçar um plano para a solução.

Basta observar que, quando retiramos duas bolas brancas, o número de bolas brancas diminui de 2. Seretiramos duas bolas de cores distintas, o número de bolas brancas não se altera. Isto significa que a paridadedo número de bolas brancas não muda ao longo de todas as retiradas de pares de bolas (Se inicialmente é par,continua par. Se inicialmente for ímpar, continua ímpar).Portanto, se p é um número par, a última bola não pode ser branca -a probabilidade é zero!Se p é um número ímpar, a última bola tem de ser branca -a probabilidade é 1!

Page 15: NOTAS DE AULA - UFRGS

2.1 Entender, antes de fazer! 15

Exemplo-8:

Dado o tabuleiro 7×7, a seguir:

6

x

4

5

pede-se para completar com números as casas vazias, de maneira tal que a soma dos números emtrês casas consecutivas (na mesma linha ou na mesma coluna) seja sempre 20.Quando completamos com números as casas vazias, que número ocupa o lugar de x?

Solução

(Olimpíada de Matemática do Perú) Depois de ler o problema (se preciso for, mais de uma vez, talvez até emvoz alta), de forma a saber exatamente quais as quantidades dadas, quais as condições dadas (hipóteses) e oque se pede, vamos traçar um plano para a solução.

Observe que o fato de que a soma dos números em três casas consecutivas seja sempre 20, significa dizerque o primeiro e o quarto desses números são iguais:

a b c d d

c

b

a

pois, temos que:a−d = (a+b+ c)− (b+ c+d) = 20−20 = 0 =⇒ a = d.

Assim, na sexta coluna podemos escrever o número 6 nas casas (6,1) e (6,4) e podemos escrever onúmero 5 nas casas (4,4) e (4,1), veja figura a seguir:

6

6

6

5

5

x

4

5

Portanto, pela hipóteses do problema, temos:

5+ x+6 = 20 =⇒ x = 9.

Page 16: NOTAS DE AULA - UFRGS

16 Capítulo 2. ESTRATÉGIAS

Exemplo-9:

Pinta-se as casas de um tabuleiro retangular alternadamente branca e preta. Em seguida, escreve-seem cada casa do tabuleiro um número inteiro. Sabe-se que a soma de todos os números escritos nascasas de uma linha e a soma dos números escritos nas casas de uma coluna é um número par.Prove que a soma de todos os números escritos nas casas pretas é um número par.

Solução

(Olimpíada de Matemática da Ucrânia-1997) Depois de ler o problema (se preciso for, mais de uma vez,talvez até em voz alta), de forma a saber exatamente quais as quantidades dadas, quais as condições dadas(hipóteses) e o que se pede, vamos traçar um plano para a solução.

Vamos supor que a casa superior esquerda esteja pintada de branco, veja figura a seguir, para o caso deum tabuleiro 8×5.

Como a soma de todos os números escritos nas casas de uma linha qualquer (ou coluna qualquer) é par,podemos concluir que a soma de todos os números escritos no tabuleiro é par. Assim, para provar o quequeremos, basta provar que a soma dos números escritas nas casas brancas seja um número par.Agora, observe que, a soma dos números escritos na primeira linha (contada de cima para baixo), maisa soma dos números escritos na terceira linha, na quinta linha etc. mais a soma dos números escritos naprimeira coluna (contada da esquerda para direita), mais a soma dos números escritos na terceira coluna, naquinta coluna etc. é igual à soma de todos os números escritos nas casas pretas mais duas vezes a soma dosnúmeros escritos em algumas casas brancas. Como, por hipótese, essa soma é par, segue que a soma dosnúmeros escritos nas casas pretas é par, como queríamos provar.

Exemplo-10:

Em torno de um círculo, escrevem-se 2016 zeros e um 1. A única operação permitida é escolherum número e mudar seus dois vizinhos, de 0 para 1 ou de 1 para 0. Aplicando a operação sucessi-vas vezes, é possível mudar todos os números para 1? E se começássemos com 2015 zeros e um 1?

Solução

(Adptada da Olimpíada de Matemática Rioplatense-1997) Depois de ler o problema (se preciso for, mais deuma vez, talvez até em voz alta), de forma a saber exatamente quais as quantidades dadas, quais ascondições dadas (hipóteses) e o que se pede, vamos traçar um plano para a solução.

É possível com 2016 zeros, mas não é possível para 2015 zeros.Observe que cada vez que se aplica a operação permitida, a paridade da soma dos números não muda.Tendo 2016 zeros, agrupamos os zeros em 504 grupos de 4, e escolhemos o primeiro e o terceiro zero decada um desses grupos para aplicar a operação.No caso de se ter 2015 zeros e um 1, é impossível começar com uma soma par e terminar com uma somaímpar no final, que é quando todos os números sejam iguais a 1.

Page 17: NOTAS DE AULA - UFRGS

2.1 Entender, antes de fazer! 17

Exemplo-11:

Na figura a seguir, temos um quadrado de lado medindo 10 cm. Calcular a área da região hachurada,sabendo-se que os pontos A, B, C e D são os pontos médios dos lados do quadrado dado.

• •

••

••

••

A

B

C

D

Solução

Depois de ler o problema (se preciso for, mais de uma vez, talvez até em voz alta), de forma a saberexatamente quais as quantidades dadas, quais as condições dadas (hipóteses) e o que se pede, vamos traçarum plano para a solução.

A idéia é juntar duas cópias da figura dada e chamar as regiões congruentes em branco de a e b, veja afigura seguir.

• •

••

••

••

••

•••

b

aa

ab

b

b

b

a

b

a

Agora, observe que juntando as regiões a e b, obtemos uma região de área igual à região hachurada.Com isso, é fácil ver o quadrado original fica dividido em 5 quadrados menores, todos congruentes aoquadrado hachurado. Isto significa dizer que a área da região hachurada é um quinto da área do quadradodado. Portanto, a área hachurada é igual a:

102

5=

1005

= 20 cm2.

Exemplo-12:

Escreve-se no quadro negro uma fileira com n sinais menos (-). Dois jogadores, A e B, disputamo jogo seguinte, em que jogam alternadamente. O jogador A começa. Uma jogada consiste em:ou substituir um dos sinais menos por um sinal mais (+) ou dois sinais menos adjacentes por doissinais mais . Aquele jogador que não puder fazer seu movimento perde. Imaginando que ambos osjogadores fazem suas melhores jogadas, o jogador A possui uma estratégia vencedora?

Solução

(ANDREESCU-SAVCHEV, página 1) Depois de ler o problema (se preciso for, mais de uma vez, talvez atéem voz alta), de forma a saber exatamente quais as quantidades dadas, quais as condições dadas (hipóteses)e o que se pede, vamos traçar um plano para a solução.O jogador A sempre vence. A estratégia do jogador consiste em:

Page 18: NOTAS DE AULA - UFRGS

18 Capítulo 2. ESTRATÉGIAS

• Se n é par.Em seu primeiro movimento, o jogador A muda para mais cada um dos dois sinais menos localizadosno centro da fileira. Então, depois dessa mudança, sempre que o jogador B fizer uma jogada, ojogador A muda para mais o(s) sinal (ais) que estejam nas posições simétrica, em relação ao centroda fileira, das aqueles mudados por B. Com isso, o jogador A fará a última jogada possível.

• Se n é ímpar.Em seu primeiro movimento, o jogador A muda para mais o sinal menos localizados no centro dafileira. Então, depois dessa mudança, sempre que o jogador B fizer uma jogada, o jogador A mudapara mais o(s) sinal (ais) que estejam nas posições simétrica, em relação ao centro da fileira, dasaqueles mudados por B. Com isso, o jogador A fará a última jogada possível.

Exemplo-13:

Seja x um número real não nulo. Sabendo-se que x+ 1x = 3, encontre x6 + 1

x6 .

Solução

Depois de ler o problema (se preciso for, mais de uma vez, talvez até em voz alta), de forma a saberexatamente quais as quantidades dadas, quais as condições dadas (hipóteses) e o que se pede, vamos traçarum plano para a solução.Temos que

x+1x= 3 =⇒

(x+

1x

)2= 32 ⇐⇒ x2 +

1x2 +2 = 9 ⇐⇒ x2 +

1x2 = 7. (∗)

Agora, de (∗), temos que(x2 +

1x2

)3= 73 ⇐⇒ x6 +

1x6 +3x4 · 1

x2 +3x2 · 1x4 = 343 ⇐⇒ x6 +

1x6 +3

(x2 +

1x2

)= 343.

Portanto, substiutindo (∗) na última expressão, temos

x6 +1x6 +3×7 = 343 ⇐⇒ x6 +

1x6 = 343−21 = 322.

Exemplo-14:

De três pessoas, A, B e C, sabe-se que algumas delas sempre falam a verdade e algumas delassempre mentem. As pessoas fazem as seguintes afirmações:• A diz que B e C sempre mentem.• B nega que seja um mentiroso.• C diz que B é um mentiroso.

Quantas dessas pessoas mentem e quantas falam a verdade?

Solução

Depois de ler o problema (se preciso for, mais de uma vez, talvez até em voz alta), de forma a saberexatamente quais as quantidades dadas, quais as condições dadas (hipóteses) e o que se pede, vamos traçarum plano para a solução.A melhor maneira de resolver o problema é examinar todas as possibilidades, a partir das três afirmaçõesfeitas. Vamos introduzir a notação: V para quem fala a verdade e M para quem mente.Agora vamos fazer uma tabela com todas as possibilidades, a partir das três afirmações feitas.

A B C1 Todos mentem M M M2 Todos falam a verdade V V V3 Dois falam a verdade V V M4 Dois falam a verdade V M V5 Dois falam a verdade M V V6 Dois mentem M M V7 Dois mentem M V M8 Dois mentem V M M

Page 19: NOTAS DE AULA - UFRGS

2.1 Entender, antes de fazer! 19

Como vimos acima, a partir das três afirmações, temos 8 possibilidades.Suponha que a possibilidade 1, ”todos mentem”, seja verdadeira. Assim, a pessoa A mente quando diz queB e C mentem. Isto significa dizer que, pelo menos uma delas fala a verdade. Logo, a possibilidade “todosmentem” é falsa.Suponha que a possibilidade 2, ”todos falam a verdade”, seja verdadeira. Assim, a pessoa C mente quandodiz que B é um mentiroso. Isto significa dizer que, pelo menos uma pessoa mente. Logo, a possibilidade“todos mentem” é falsa.De modo análogo, é fácil ver que as possibilidades 3, 4, 5 e 7 são falsas.Assim, as possibilidades reais de acontecer são as possibilidades 6 e 8. Portanto, existem duas pessoa quementem e uma que fala a verdade.

Exemplo-15:

Bolas de tênis de mesmo raio r são hermeticamente embaladas em latas cilíndricas com 3 bolas,onde elas apenas tocam a lata na superfície lateral, no topo e no fundo, veja figura a seguir.

3d

d

d

d

Qual é maior: a altura da lata embalagem ou a medida da circunferência de cada bola?

Solução

Depois de ler o problema (se preciso for, mais de uma vez, talvez até em voz alta), de forma a saberexatamente quais as quantidades dadas, quais as condições dadas (hipóteses) e o que se pede, vamos traçarum plano para a solução.A resposta é: a medida da circunferência de cada bola.Observe que a altura da lata é igual a 3d, onde d é o diâmetro da circunferência de cada bola. A medida dacircunferência de cada bola é igual 2πr = 2π

d2 = πd > 3d, pois π = 3,14 · · ·> 3.

Exemplo-16:

João colocou suas cabras num caminhão para vender na feira. Também pôs no caminhão umaquantidade de repolhos exatamente igual ao quadrado do número de cabras. Durante a viagem, cadacabra comeu dois repolhos. Na feira, João vendeu 5 cabras e um número de repolhos. No final dodia, ele observou com surpresa que o número de repolhos que tinha era igual ao quadrado o númerode cabras que tinha sobrado. Em seguida, colocou tudo o que não vendeu no caminhão e retornou.Mas durante a viagem, cada cabra comeu dois repolhos e chegando em sua casa, João tinha cabrasmas não mais repolho. Quantos repolhos João vendeu na feira?

Solução

Depois de ler o problema (se preciso for, mais de uma vez, talvez até em voz alta), de forma a saberexatamente quais as quantidades dadas, quais as condições dadas (hipóteses) e o que se pede, vamos traçarum plano para a solução.A resposta é 31.Seja x a quantidade de cabras que João colocou no caminhão. Assim, pela hipótese, concluímos que Joãocolocou x2 repolhos no caminhão.Ao chegar ao mercado, sobraram x2−2x repolhos. Se ele vendeu y repolhos sobraram x2−2x− y. Mas,como João vendeu 5 cabras, sobraram x−5 cabras.

Page 20: NOTAS DE AULA - UFRGS

20 Capítulo 2. ESTRATÉGIAS

Assim, temosx2−2x− y = (x−5)2 (∗)

Durante a viagem de volta os x2−2x− y repolhos foram comidos por x−5 cabras, que, por hipóteses,comiam na razão de 2 por cabra.Logo, temos:

x2−2x− y = 2(x−5) (∗∗)

De (∗) e (∗∗), podemos concluir que

(x−5)2 = 2(x−5), com,(x−5) 6= 0.

Portanto, x−5 = 2 o que implica x = 7, que é o número de cabras. Substituindo esse valor em (∗),obtemos:

72−2 ·7− y = (7−5)2 =⇒ 49−14− y = 4 =⇒ y = 31

.

Page 21: NOTAS DE AULA - UFRGS

2.2 A busca do plano 21

2.2 A busca do plano

Uma das técnicas de criatividade, chamada tempestade de ideias (em inglês brainstorming), geralmentefeita em grupo, revela que a quantidade gera a qualidade. O método foi popularizado nos anos de 1930pelo americano Alex Faicney Osborn. A fase em que você procura um plano capaz de levar a solução de umproblema é uma situação parecida com a tempestade de ideia, só que exercida muitas vezes solitariamentepor você. Esta fase do processo de resolução do problema é aquela em que deve nascer da sua cabeçamuitas ideias, mesmo que possam parecer totalmente inócuas.

Durante este processo, nenhuma ideia deve ser descartada ou julgada como errada ou absurda. As vezes, asideias em que você menos aposta podem se revelar as mais apropriadas. Por isso, anote todas as suas ideias.

Se você já tem algumas estratégias possíveis para atacar o problema, a tarefa a seguir é meditar sobre assuas ideias e deixar que o seu subconsciente trabalhe, a seu gosto, esse amontoado de ideias que vocêpreparou para ele. É aconselhável que você faça uma lista das melhores ideias, a princípio sem misturá-las.Explore cada ideia da lista com decisão e confiança, de forma ordenada, em paz, sem precipitações.

Se, ao colocar em prática uma ideia, lhe ocorrer outra, totalmente desligada da primeira, e que você avaliaque pode lhe ajudar, não vá desprezá-la. Coloque-a na sua lista. Mas, também não desvie sua atenção daque agora está explorando.

Um ponto importante que deve sempre ser lembrado: você não pode desistir facilmente. Por outro lado, nãodeve teimar demais só com uma ideia. Se as coisas complicarem demais, haverá provavelmente outrocaminho. Um caminho que foi muito usado na história de matemática foi o da tentativa e erro, tentativa eerro, tentativa e erro, você não pode esquecer este fato.

Ao concluir sua resolução, você precisa ter certeza disso. Reveja sua solução com cuidado.

Uma verdade: as meias ideias e as meias soluções de pouco servem! É preciso certificar de que, realmente,você chegou à solução.

Se você conseguir resolver o problema, ótimo.

Se trabalhou horas a fio e não conseguiu vislumbrar uma solução, não se preocupe. Muitas vezes se aprendeprofundamente com os problemas que se tenta, com interesse, determinação e persistência ...... e não seconsegue resolver, do que com os que se resolve à primeira vista.

Um conselho deve ser lembrado: é mais importante a qualidade do que a quantidade. Não se apresse emdemasia. Ao concluir a solução de um problema, é preciso que você reflita sobre o processo, para que tenhauma ideia das dificuldades, dos becos sem saídas em que se meteu e, principalmente, como deve proceder nofuturo para resolver melhor outros problemas, parecidos ou não.

Uma reflexão sobe o nosso próprio processo de pensamento é interessante na medida em que podemos tirarbons proveitos para o futuro. Cada um tem seu próprio estilo de conhecimento. Visual ou analítico?Depende muito da expressão verbal ou da forma escrita? Tem tendência para o compromisso com ideiaúnica, sem flexibilidade? Tem tendência a pensar em círculo obsessivamente? Como se pode fomentar ofluxo de ideias novas, variadas, originais? Reflexões como essas ajudam, a saber, que tipo de problemasvocê pode se ocupar com sucesso e em quais deles sua probabilidade de êxito não é tão grande.

Exemplo-17:

Dois jogadores, A e B, disputam um jogo num tabuleiro 10× 10, jogando alternadamente. Ojogador A possui uma lata de tinta azul e o jogador B uma lata de uma tinta vermelha. O jogadorA começa. Na sua vez de jogar, cada jogador escolhe uma linha ou coluna do tabuleiro que nãotenha sido previamente escolhida por qualquer um dos dois e pinta suas 10 casas com sua própriacor. Se qualquer uma destas casas escolhidas já estava pintada, a nova cor cobre a anterior. Após20 jogadas, quando não há mais as linhas e colunas disponíveis, o jogo termina. Então conta-seo número de casas de cada cor e determina-se o vencedor de acordo com a seguinte regra: se onúmero de casas vermelhas exceder a dez mais do que a quantidade de casas azuis, então o jogadorB vence. Caso contrário, o jogador A ganha.Determinar se um dos dois jogadores tem uma estratégia vencedora e explicar a estratégia.

Page 22: NOTAS DE AULA - UFRGS

22 Capítulo 2. ESTRATÉGIAS

Solução

Depois de ler o problema (se preciso for, mais de uma vez, talvez até em voz alta), de forma a saberexatamente quais as quantidades dadas, quais as condições dadas (hipóteses) e o que se pede, vamos traçarum plano para a solução.

Veremos, a seguir, que o jogador B possui uma estratégia vencedora. A estratégia do jogador B consisteem, cada vez que o jogador A fizer seu movimento, ele escolhe uma linha ou coluna perpendicular àescolhida por A no movimento anterior. Para provar que esta estratégia é vencedora, basta observar quecada casa do tabuleiro é pintada exatamente duas vezes, uma quando um jogador escolhe a linha em queestá localizada a casa e a outra quando um escolhe a coluna. Assim, após as duas primeiras jogadas umaúnica casa fica com sua cor final e esta é vermelha. Depois da terceira e quarta jogadas, três novas caixasatingem sua cor final, e das quais duas serão vermelhas e uma será azul. Em geral, após o movimentos2k−1 e 2k existem 2k−1 novas casa que atingiram suas cores finais, das quais k−1 serão azuis e kvão ser vermelhas. Como a cada duas jogadas a quantidade de casas vermelhas aumenta em um, ao final ojogador B terá uma vantagem de 10 casas sobre o jogador A, vencendo o jogo.

Exemplo-18:

Dois jogadores, A e B, disputam o seguinte jogo, em que jogam alternadamente, usando um maçode 2002 cartões. Cada cartão tem nele escrito um números inteiro de 1 a 2002 (um só númeroem cada cartão). Colocam-se os cartões sobre uma mesa, com a face virada para cima (os doisjogadores vendo o número em cada cartão). O jogador A começa. Uma jogada consiste em pegarum cartão da mesa, até que todos os cartões sejam retirados. Depois de retirados todos os cartões,cada jogador soma os números de seus cartões. Vence o jogo aquele jogador para o qual o últimodígito da soma de todos os números de seus cartões é maior que o último dígito da soma de todos osnúmeros dos cartões do seu adversário. Imaginando que ambos os jogadores fazem suas melhoresjogadas, quem possui uma estratégia vencedora: A ou B?

Solução

(Tournament of the Towns, Junior O level, fall 2002). Depois de ler o problema (se preciso for, mais de umavez, talvez até em voz alta), de forma a saber exatamente quais as quantidades dadas, quais as condiçõesdadas (hipóteses) e o que se pede, vamos traçar um plano para a solução.

Nosso plano de solução vai ser a estratégia do jogador A, que sempre vence. A estratégia do jogador Aconsiste em formar todos os pares de cartões numerados com (k,1000+ k), com k = 1,2,3, . . . ,1000,mais o par (2001,2002).Agora, observe que todos os 1000 pares são formados com números que possuem o mesmo último dígito (omesmo dígito das unidades).Na sua primeira jogada, o jogador A começa escolhendo para pegar o cartão numerado com 2002. Apartir deste momento, sua estratégia é buscar o outro elemento do par escolhido a cada vez que o jogadorB fizer sua escolha. Então, eventualmente, o segundo jogador é forçado a pegar em algum momento ocartão numerado com 2001.Em geral, se ainda tem carta não retiradas e todas as cartas escolhidas formam pares do tipo (k,1000+ k),na sua próxima jogada o jogador A retira qualquer carta numerada, digamos com k′. Em seguida, osegundo jogador terá que pegar, em algum momento, o cartão com o qual formará o par com o cartãonumerado com k′. Isto decorre do fato de que, cada vez que o jogador B seleciona algum cartão, o jogadorA escolhe o cartão que forma o par com aquele escolhido por B.No final do jogo, a soma dos cartões escolhidos pelo jogador A será igual a

1+2+3+ . . .+1000+2002 =1000 ·1001

2+2002 = 500 ·1001+2002,

que tem como último dígito o 2, enquanto a soma dos números dos cartões escolhido polo jogador Btermina em 1, o que garante a vitória de A.

Page 23: NOTAS DE AULA - UFRGS

2.2 A busca do plano 23

Exemplo-19:

Dois jogadores, A e B, disputam um jogo em que jogam alternadamente. De um baralho de cartasextrai-se nove cartas numeradas de 2 a 10 e as coloca com a face para cima sobre a mesa. Umajogada consiste em retirar uma das cartas. O jogador A começa. Vence quem primeiro conseguetrês cartas cuja soma seja exatamente 18. Quem possui uma estratégia para não perder: A ou B?

Solução

Depois de ler o problema (se preciso for, mais de uma vez, talvez até em voz alta), de forma a saberexatamente quais as quantidades dadas, quais as condições dadas (hipóteses) e o que se pede, vamos traçarum plano para a solução.

O jogador A possui uma estratégia para não perder o jogo. Sua estratégia consiste em escrever os novenúmeros como segue, formando um quadrado mágico, onde a soma dos números em qualquer linha,qualquer coluna e qualquer diagonal seja igual a 18:

7

8

3

2

6

10

9

4

5

Com esta estratégia, o jogador A joga como se estivesse disputando uma partida do jogo da velha. 1 Sena sua primeira jogada ele escolhe a carta de número 6, a carta que está no centro do tabuleiro, ele nãoperde o jogo, basta seguir atentamente a estratégia do jogo da velha.

Exemplo-20:

O número N é o produto de k números primos distintos (k≥ 3). Dois jogadores, A e B, disputamo jogo seguinte em que jogam alternadamente. O jogador A começa. Uma jogada consiste emescrever no quadro-negro um divisor composto de N. Nenhum dos jogadores pode escrever N.Também nunca pode aparecer dois números relativamente primos ou dois números, sendo um divisordo outro. O primeiro jogador que não puder fazer seu movimento perde. Quem tem uma extratégiavencedora: A ou B?

Solução(Olimpíada de Matemática de São Petersburg - 1997) Depois de ler o problema (se preciso for, mais de umavez, talvez até em voz alta), de forma a saber exatamente quais as quantidades dadas, quais as condiçõesdadas (hipóteses) e o que se pede, vamos traçar um plano para a solução.O jogador A possui uma estratégia vencedora. A estratégia do jogador A consiste em, no seu primeiromovimento, escolher o número pq, com p, q números primos distintos. Todo número subsequente terá de

1Jogo da Velha O jogo da velha ou jogo do galo (no português europeu) é um jogo e passatempo popular. É um jogo de regrasextremamente simples, que não traz grandes dificuldades para seus jogadores e é facilmente aprendido. Seu nome teria se originadona Inglaterra, quando nos finais das tardes, mulheres se reuniram para conversar e bordar. As mulheres idosas, por não terem maiscondições de bordar em razão da fraqueza da visão, jogavam este jogo simples, que passou a ser conhecido como o da "velha". Porém,sua origem teria sido ainda mais antiga. As regras do jogo são:

• O tabuleiro é uma matriz de três linhas por três colunas.• Dois jogadores escolhem uma marcação cada um, geralmente um círculo (O) e um xis (X).• Os jogadores jogam com as mãos uma marcação por vez em uma lacuna que esteja vazia.• O objetivo é conseguir 3 círculos ou três xis em linha horizontal, vertical ou diagonal, e ao mesmo tempo, quando possível,

impedir o adversário de ganhar na próxima jogada.• Quando um jogador conquista o objetivo, costuma-se riscar os três símbolos.• Quando um dos jogadores vence, o mesmo começa a outra partida

. (Referência: htt ps : //pt.wikipedia.org/wiki/Jogodavelha)

Page 24: NOTAS DE AULA - UFRGS

24 Capítulo 2. ESTRATÉGIAS

ser da forma pn ou pn, para algum número inteiro n > 1, relativamente primo com pq. Se o jogador Bescreve no quadro-negro pn, o jogador A escreve qn, e vice-versa. Com isso, o jogador A vence.

Exemplo-21:

Divide-se um tabuleiro de xadrez 8×8, veja figura abaixo, em K retângulos que não se sobrepõem,de acordo com as seguintes regras:• Cada retângulo é formado unicamente juntando casas do tabuleiro (quadrados 1×1).• Cada retângulo possui a mesma quantidade de casas brancas e pretas.• Não existem dois retângulos que sejam formados pela mesma quantidade de casas.

Qual é o maior valor possível para K?

Solução

(Olimpíada de Matemática do Perú) Depois de ler o problema (se preciso for, mais de uma vez, talvez até emvoz alta), de forma a saber exatamente quais as quantidades dadas, quais as condições dadas (hipóteses) eo que se pede, vamos traçar um plano para a solução.

Antes de estabelecer o plano para a solução, observe que, pela segunda condição do problema, as áreaspossíveis dos K retângulos devem ser números pares e, pela terceira condição, todos os números paresdevem ser distintos.Assim, se S é a soma das áreas dos K retângulos, S deve ser maior do que ou igual a soma dos primeirosK números pares positivos. Além disso, S é no máximo igual a 64, que é quantidade de casas dotabuleiro. Logo, temos:

2+4+6+ . . .+2K ≤ 64 ⇐⇒ 2K(K +1)2

≤ 64 ⇐⇒ K(K +1)≤ 64 =⇒ K ≤ 7.

Para concluir, basta mostrar que podemos realizar a divisão do tabuleiro com exatamente 7 retângulossatisfazendo as condições do problema.De fato, veja a figura abaixo, na qual cada número representa a quantidade de casas de cada retângulo:

2 14

4 12

6 10

16

Page 25: NOTAS DE AULA - UFRGS

2.2 A busca do plano 25

Exemplo-22:

Encontre todos os pares de números inteiros positivos (x,y), para os quais

1+1996x+1998y = xy.

Solução

(Olimpíada de Matemática da Irlanda) Depois de ler o problema (se preciso for, mais de uma vez, talvez atéem voz alta), de forma a saber exatamente quais as quantidades dadas, quais as condições dadas (hipóteses)e o que se pede, vamos traçar um plano para a solução.

Vamos reescrever a expressão dada de uma forma equivalente para que possamos encontrar um plano quenos leve à solução do problema.:

1+1996x+1998y= xy ⇐⇒ xy−1996x−1998y= 1 ⇐⇒ xy−1996x−1998y+1996·1998= 1+1996·1998 ⇐⇒

⇐⇒ (x−1998) · (y−1996) = 1+(1997−1) · (1997+1) ⇐⇒ (x−1998) · (y−1996) = 19972.

Agora, observe que o número 1997 é primo. Assim, temos, obrigatoriamente, que ter:

x−1998 =±1, ±1997, ±19972 e y−1996 =±1, ±1997, ±19972,

o que nos leva às seis soluções possíveis:

(1999,19972+1996), (1997,−19972+1996), (3995,3993), (1,−1), (19972+1998,1997), (−19972+1998,1995).

Exemplo-23:

Prove que entre quaisquer dez pontos localizados na região limitada por um círculo de diâmetroigual 5, existem dois desses pontos que estão distanciados um do outro por no máximo 2.

Solução

(Olimpíada de Matemática do Japão) Depois de ler o problema (se preciso for, mais de uma vez, talvez atéem voz alta), de forma a saber exatamente quais as quantidades dadas, quais as condições dadas (hipóteses)e o que se pede, vamos traçar um plano para a solução.

Divida a região limitada pelo círculo dado em nove partes: 1 círculo concêntrico de raio 1 e mais 8partes formadas pelas sub-regiões determinadas pela parte de cada um dos setores congruentes que estejafora da região limitada pelo círculo concêntrico de raio 1, veja figura a seguir.

Agora, é fácil ver que, dados 10 pontos localizados na região limitada por um círculo de diâmetro igual 5,existem dois desses pontos dentro de uma mesma (destas nove) sub-regiões e estes dois pontos estão a umadistância de no máximo 2, o que conclui a prova.

Page 26: NOTAS DE AULA - UFRGS

26 Capítulo 2. ESTRATÉGIAS

Exemplo-24:

Escrevem-se em linha reta os números inteiros de 1 a 37, de modo que cada número divide a somados números anteriormente escritos. Se o primeiro número escrito é 37 e o segundo é o 1, qual é oterceiro número escrito?

Solução

(Olimpíada de Matemática da Rússia) Depois de ler o problema (se preciso for, mais de uma vez, talvez atéem voz alta), de forma a saber exatamente quais as quantidades dadas, quais as condições dadas (hipóteses)e o que se pede, vamos traçar um plano para a solução.

Observe que, pela hipótese do problema o terceiro número é um divisor de 37+1 = 38 = 2×19. Logo, oterceiro número escrito pode ser 2 ou 19. Agora, para dirimir a dúvida, o segredo é olhar para o últimonúmero escrito: x. Pela hipótese do problema, temos que x divide a expressão:

1+2+3+ . . .+37 =37 · (37+1)

2= 37×19.

Como 37 e 19 são números primos e 1 < x < 37, segue que x = 19. Logo, o terceiro número não podeser 19. Portanto, o terceiro número tem de ser 2.

Exemplo-25:

Divide-se em triângulos a região limitada por um polígono regular de 1997 lados, usando diagonaisque não se interceptam no interior da região. Prove que no mínimo um destes triângulos é acutângulo.

Solução

(Olimpíada de Matemática da Rússia) Depois de ler o problema (se preciso for, mais de uma vez, talvez atéem voz alta), de forma a saber exatamente quais as quantidades dadas, quais as condições dadas (hipóteses)e o que se pede, vamos traçar um plano para a solução.

Basta observar que o círculo circunscrito ao polígono dado é também o círculo circunscrito a cada um dostriângulos. Como o polígono possui um número ímpar de lados, segue que o centro do círculo circunscritonão pertence a qualquer uma das diagonais. Isto é, o centro do círculo está localizado no interior de um dostriângulos. Isto é suficiente para que possamos concluir que o triângulo é acutângulo.

Exemplo-26:

Escrevem-se num quadro negro os números inteiros de 1 até 1000. Dois jogadores, A e B,disputam o jogo seguinte, em que fazem seus movimentos alternadamente. O jogador A começa.Uma jogada consiste em apagar um dos números escritos no quadro. O jogo termina quandorestarem dois números. O jogador A vence se a soma dos dois números restantes for divisível por3, caso contrário, o jogador B ganha. Qual dos dois jogadores possui uma estratégia vencedora?

Solução

(Olimpíada de Matemática da Rússia) Depois de ler o problema (se preciso for, mais de uma vez, talvez atéem voz alta), de forma a saber exatamente quais as quantidades dadas, quais as condições dadas (hipóteses)e o que se pede, vamos traçar um plano para a solução.

O jogador B possui uma estratégia vencedora: se o jogador A apaga o número x, então o jogador Bapaga o número 1001− x. Portanto, os últimos dois números somarão 1001, que não é divisível por 3.

Page 27: NOTAS DE AULA - UFRGS

2.2 A busca do plano 27

Exemplo-27:

Escreve-se um número em cada uma das 16 casas de um tabuleiro 4×4 . Para qualquer casa, asoma dos números escritos nas casas que tem um lado comum com ela é igual a 1. Determine asoma dos 16 números escritos nas casas do tabuleiro.

Solução

(Torneio das Cidades - 2000) A resposta é 6. Depois de ler o problema (se preciso for, mais de uma vez,talvez até em voz alta), de forma a saber exatamente quais as quantidades dadas, quais as condições dadas(hipóteses) e o que se pede, vamos traçar um plano para a solução.

A estratégia é pintar as casas do tabuleiro alternadamente branca e marrom. Cada casa branca é vizinha,com um lado comum, de uma das casas marrons assinaladas, veja figura a seguir.

X

X

X

Observe que, pela hipótese do problema, podemos concluir que a soma dos números nas casas brancas éigual a 3. Resta mostrar que a soma dos números nas casa marrons é igual a 3. Para isso, preencha ocentro do tabuleiro com 0 e as demais casas como na figura a seguir.

z

y

x

w

w

0

0

z y

0

0

x

x

w

z

y

Como x+ z = 1 e y+w = 1, segue que a soma dos números escritos nas casas do tabuleiro é igual a

3(x+ z)+3(y+w) = 3 ·1+3 ·1 = 3+3 = 6.

Exemplo-28:

Sejam n um número inteiro positivo. Divide-se um tabuleiro retangular 4× n em retân-gulos 2× 1 ou 1× 2 (como se o tabuleiro fosse coberto com peças de um dominó, semsuperposições e sem deixar buracos, com cada dominó cobrindo precisamente duas casas dotabuleiro). Em seguida, pinta-se de vermelho todos os pontos do tabuleiro que são vértices de algumdos retângulos 2×1 ou 1×2. Qual é a menor quantidade de pontos vermelhos que se pode obter?

Solução

(Olimpíada de Matemática do Perú) Depois de ler o problema (se preciso for, mais de uma vez, talvez até emvoz alta), de forma a saber exatamente quais as quantidades dadas, quais as condições dadas (hipóteses) eo que se pede, vamos traçar um plano para a solução.

Page 28: NOTAS DE AULA - UFRGS

28 Capítulo 2. ESTRATÉGIAS

Observe que, como n é um inteiro positivo qualquer, nossa resposta vai ser dada em função de n. Assim,vamos chamar de f (n) o número mínimo de pontos vermelhos que podemos obter ao cobrir um tabuleiro4×n com peças de um dominó, satisfazendo às hipóteses do problema. Fazendo um desenho, é fácil verque:

f (1) = 6, f (2) = 9, f (3) = 12, f (4) = 15 e f (5) = 18.

• • •

• • •n = 1 f(1) = 6

• • •

• • •

• • •

n = 2 f(2) = 9

• • •

• • •

• • •

• •

n = 3 f(3) = 12

• • • • •

• • • • •

• • • • •

n = 4 f(4) = 15

• • • • •

• • • • •

• • • • •

• • •

n = 5 f(5) = 18

Nosso objetivo é mostra por indução que:

f (2k) = 5k+2 e f (2k+1) = 5k+8, para todo nmero inteiro k ≥ 2. (∗)

Para isso, seja n≥ 2, e suponha que já conhecemos os valores de f (1), f (2), f (3), . . . , f (2n), f (2n+1).Agora, considere um tabuleiro 4× (2n+2) coberto completamente por dominós, nas hipóteses doproblema. Podem ocorrer três situações:

Page 29: NOTAS DE AULA - UFRGS

2.2 A busca do plano 29

• A última linha do tabuleiro, contada de baixo para cima, possui como cobertura dois dominóscolocados horizontalmente.

• Nas duas últimas linhas, contada de baixo par a cima, existem 4 peças de dominós colocadasverticalmente.

• Nas duas últimas linhas do tabuleiro, contadas de baixo para cima, hajam 1 peça de dominócolocada horizontalmente 2 colocadas verticalmente.

Vamos provar nosso objetivo (∗) para cada um dos casos acima.

• A última linha do tabuleiro, contada de baixo para cima, possui como cobertura dois dominóscolocados horizontalmente, veja figura a seguir.

......

......

......

......

2n + 1

• • •

Portanto, esta última linha, contada de baixo para cima, gera 3 novos pontos vermelhos, o quesignifica que existem pelo menos

3+ f (2n+1) = 3+5n+8 = 5n+11 pontos vermelhos.

• Nas duas últimas linhas, contada de baixo par a cima, existem 4 peças de dominós colocadasverticalmente, veja figura ilustrativa a seguir .

Page 30: NOTAS DE AULA - UFRGS

30 Capítulo 2. ESTRATÉGIAS

......

......

......

......

2n

• • • • •

• • • • •

Portanto, estas duas últimas linhas, contada de baixo para cima, geram 5 novos pontos vermelhos, oque significa que existem pelo menos

5+ f (2n) = 5+5n+5 = 5n+10 pontos vermelhos.

• Nas duas últimas linhas do tabuleiro, contadas de baixo para cima, hajam 1 peça de dominócolocada horizontalmente 2 colocadas verticalmente, veja figuras ilustrativas a seguir.

......

......

......

......

• • •

• • •

• •

• •

• • •

• • • •

2n + 2 - k

k

Assim, teremos um total de pontos vermelhos no mínimo igual a:

4+3(k−1)+ f (2k+2− k) = 5n+11+k2, se k é par ou 5n+11+

k+12

, se k é ímpar.

Page 31: NOTAS DE AULA - UFRGS

2.2 A busca do plano 31

Observe que ambos os valores são maiores do que 5n+10.Com isso, concluímos que f (2n+2)≤ 5n+10, que é o menor obtido ao analisarmos os dois casos.Colocando todas peças do dominó na horizontal é fácil ver que existem um total de 5n+10 pontosvermelhos. Portanto, f (2n+2) = 5n+10.De forma análoga, é fácil ver que f (2n+3) = 5n+13, o que completa a prova.

Exemplo-29:

Diga, justificando, se é possível escolher 1000 pontos do plano de tal modo que no mínimo 6000distâncias entre dois deles sejam iguais.

Solução

(BW - 2003) Depois de ler o problema (se preciso for, mais de uma vez, talvez até em voz alta), de forma asaber exatamente quais as quantidades dadas, quais as condições dadas (hipóteses) e o que se pede, vamostraçar um plano para a solução.

A resposta é sim. Comece com uma configuração de 4 pontos e 5 distâncias, veja figura a seguir.

dd

Agora tome a figura acima e mais duas cópias congruentes a ela, obtidas por movimentos paralelos (e nãocoincidentes), veja figura a seguir.

dd

dd

d d

Neste caso, tem-se 3×4 = 12 pontos e 3×5+12 = 27 distâncias. Agora junte mais três cópias da figuraacima, obtendo:

3×12 = 36 pontos e 3×27+36 = 117 distancias.

Agora, forme uma figura com 9 figuras congruentes a figura inicial, obtidas dela por movimentos paralelos(e não coincidentes). Desse modo, temos

3×36 = 108 pontos e 3×117+108 = 459 distancias.

Page 32: NOTAS DE AULA - UFRGS

32 Capítulo 2. ESTRATÉGIAS

Procedendo desta maneira, sempre juntando mais 3 cópias da figura inicial, vamos obtendo gradativamente:

3×108 = 324 pontos e 3×459+324 = 1701 distancias.

3×324 = 972 pontos e 3×1701+972 = 6075 distancias.

Page 33: NOTAS DE AULA - UFRGS

2.3 Procure semelhanças com outros problemas 33

2.3 Procure semelhanças com outros problemasComo não há nada de novo debaixo do céu, é conveniente procurar semelhanças do problema dado comoutros que você já conhece.

Pergunte a si mesmo: o que é que esse problema me faz lembrar?

Se seu problema for genérico, tente primeiro alguns casos particulares. Caso o problema envolva ageometria tridimensional, você poderá tentar primeiro um problema bidimensional análogo.

Exemplo-30:

Resolva a equação x4−5x2 +6 = 0.

Solução

Depois de ler o problema (se preciso for, mais de uma vez, talvez até em voz alta), de forma a saberexatamente quais as quantidades dadas, quais as condições dadas (hipóteses) e o que se pede, vamos traçarum plano para a solução.

Se a equação dada parece difícil, troque x2 por t, obtendo a equação

t2−5t +6 = 0 (∗),

que é mais fácil de resolver. Assim, as soluções da equação (∗) são t = 2 e t = 3, o que implica x2 = 2ou x2 = 3. Portanto, as soluções da equação dada são: ±

√2 e ±

√3.

Exemplo-31:

No vértice A de uma caixa retangular fechada temos uma formiga e no vértice B temos outraformiga, veja Figura a seguir.

A formiga na posição A vai se deslocar sobre a caixa para encontrar a formiga na posição B.Qual é o caminho mais curto sobre a caixa, ligando o ponto A ao ponto B, que a formiga deveescolher para fazer seu deslocamento?

Solução

Depois de ler o problema (se preciso for, mais de uma vez, talvez até em voz alta), de forma a saberexatamente quais as quantidades dadas, quais as condições dadas (hipóteses) e o que se pede, vamos traçarum plano para a solução.

O que é que esse problema me faz lembrar?

Se a questão fosse num retângulo, figura plana, em vez de uma caixa retangular, figura tridimensional, aquestão seria resolvida simplesmente procurando o menor caminho sobre o plano ligando dois pontos, quesabemos ser um segmento de reta. O que temos que fazer é tornar a caixa tridimensional um objetobidimensional. Como fazer isso? Abrindo a caixa, marcando os dois pontos e traçando o segmento de retaligando os pontos A e B, veja figura a seguir.

Page 34: NOTAS DE AULA - UFRGS

34 Capítulo 2. ESTRATÉGIAS

A

B

Depois disso, recompomos a caixa, mostrando o menor caminho ligando os pontos A e B, veja figura aseguir.

Exemplo-32:

Qual é a soma dos n termos da séries 7+77+777+7777+ · · ·?

Solução

Depois de ler o problema (se preciso for, mais de uma vez, talvez até em voz alta), de forma a saberexatamente quais as quantidades dadas, quais as condições dadas (hipóteses) e o que se pede, vamos traçarum plano para a solução.

Se em vez de 7+77+777+7777+ · · · tivéssemos

9+99+999+9999+ · · ·?

Neste caso, reescreveríamos a séries como:

9+99+999+9999+ · · ·= (10−1)+(100−1)+(1000−1)+(10000−1)+ · · ·=

= (10+100+1000+10000+ · · ·)−n

As potências de 10 formam uma progressão geométrica de n termos cuja soma é igual a 10n+1−109 . Assim,

9+99+999+9999+ · · ·=

= (10−1)+(100−1)+(1000−1)+(10000−1)+ · · ·= 10n+1−109

−n.

Agora, observe que para encontrar a soma pedida, basta observar que 7 é igual a 79 de 9, o que implica que

a soma pedida é igual a

7+77+777+7777+ · · ·= 7(1+11+111+1111+ · · ·) = 79×9(1+11+111+1111+ · · ·) =

Page 35: NOTAS DE AULA - UFRGS

2.3 Procure semelhanças com outros problemas 35

=79(9+99+999+9999+ · · ·) =

[10n+1−10

9−n]=

781

(10n+1−9n−10).

Exemplo-33:

Numa sala de aula existem 25 alunos sentados em 5 filas com 5 lugares em cada fila. Um dia oprofessor pede para cada aluno mudar seu lugar como segue: cada um deve mover para um assentopara a frente ou para trás ou um assento à esquerda ou à direita - movimento em diagonal não épermitido.É possível que todos os 25 alunos sigam estas instruções?

Solução

Depois de ler o problema (se preciso for, mais de uma vez, talvez até em voz alta), de forma a saberexatamente quais as quantidades dadas, quais as condições dadas (hipóteses) e o que se pede, vamos traçarum plano para a solução.

A abordagem tradicional é tentar vários movimentos. Isso geralmente não resolve o problema e causa certafrustração. A ideia aqui é resolver um problema análogo mais simples. Assim, faça um desenho da sala enumere os assentos, veja figura a seguir.

1 2 3 4 5

6 7 8 9 10

11 12 13 14 15

16 17 18 19 20

21 22 23 24 25

Agora, observe que, dentre os números inteiros de 1 a 25, temos 13 números ímpares e somente 12números pares. Com os movimentos permitidos, cada aluno passa de uma cadeira de número ímpar parauma cadeira de número par, e vice-versa. Como a quantidade de números pares difere da quantidade denúmeros ímpares, vai haver um aluno que que não pode se movimentar.Portanto, é impossível que todos os 25 alunos sigam as instruções do professor.

Exemplo-34:

Encontre o valor da expressão:

S = 1002−992 +982−972 +962−952 + . . .+42−32 +22−12. (∗)

Solução

Depois de ler o problema (se preciso for, mais de uma vez, talvez até em voz alta), de forma a saberexatamente quais as quantidades dadas, quais as condições dadas (hipóteses) e o que se pede, vamos traçarum plano para a solução.

Não é confortável efetuar as operações indicação, pela quantidade de números existente. Em vez disso,usaremos a expressão para a diferença de dois quadrados:

x2− y2 = (x+ y) · (x− y)

Page 36: NOTAS DE AULA - UFRGS

36 Capítulo 2. ESTRATÉGIAS

Assim, a soma (∗) pode ser escrita como:

S = (100+99) ·(100−99)+(98+97) ·(98−97)+ . . .+(4+3) ·(4−3)+(2+1) · · ·(2−1). (∗∗)

Agora, observe que todas as expressões do segundo parênteses de cada parcela de S em (∗∗) é igual a 1.Portanto, temos S = 100+99+98+97+ . . .+4+3+2+1 = 100(100+1

2 = 50×101 = 5050.

Exemplo-35:

Para que valores de n o número 28 +211 +2n é um quadrado perfeito?

Solução

(Olimpíada de Matemática do Cone Sul) Depois de ler o problema (se preciso for, mais de uma vez, talvezaté em voz alta), de forma a saber exatamente quais as quantidades dadas, quais as condições dadas(hipóteses) e o que se pede, vamos traçar um plano para a solução.

Podemos observar que:28 +211 +2n = (24)2 +2 ·24 ·26 +2n

Agora, observe que o número do lado direito da igualdade acima é muito parecida com a expressão doquadrado de um binômio:

(a+b)2 = a2 +2ab+b2.

Assim, tomando a = 24, b = 26, teremos: (24 +26)2 = (24)2 +2 ·24 ·26 +212, o que nos leva a concluirque n = 12.Resta-nos a pergunta: n = 12 é a única solução?A resposta é sim. Suponhamos que exista outra solução. Assim, suponha que existe um número inteiro kpara o qual

28 +211 +2n = k2.

Neste caso, podemos escrever

28+211+2n = 28+23 ·28+2n = k2 ⇔ 9 ·28+2n = k2 ⇔ 2n = k2−32 ·28 ⇔ 2n = (k+3 ·24)(k−3 ·24).

Assim, é claro que os fatores (k+3 ·24) e (k+3 ·24) são obrigatoriamente potência de 2. Portanto,existem números naturais a e b, com a+b = n, tais que

2a = k+3 ·24 e 2b = k−3 ·24.

Logo, 2a−2b = 2 ·3 ·24⇐⇒ 2b · (2a−b−1) = 3 ·25.Portanto, 2b = 25 e 2a−b−1 = 3, o que resulta em b = 5, 2a−b = 4⇒ a = 7. Portanto, n = a+b = 12,o que conclui a prova.

Page 37: NOTAS DE AULA - UFRGS

2.4 Começar pelo fácil torna fácil o difícil 37

2.4 Começar pelo fácil torna fácil o difícilTalvez o problema seja complicado porque há muitos elementos. Tente tornar o problema mais fácil.Construa um, menos complicado, com menos dados. Talvez essa situação venha lhe revelar algo que facilitea solução do problema mais complexo.

Exemplo-36:

Prove que, se a, b,c são números reais, então

a2b2 +b2c2 +a2c2 ≥ abc(a+b+ c)

Solução

Em vez de resolver a questão diretamente, é mais confortável pensar no problema seguinte, que é massimples e equivalente ao problema que queremos resolver:

“Se x, y, z são números reais, prove que:

x2 + y2 + z2 ≥ xy+ xz+ yz′′.

É fácil ver que, a solução do nosso problema mais simples decorre do fato de que a soma de três quadradosperfeitos é sempre maior do que ou igual a zero:

(x−y)2+(y−z)2+(x−z)2 ≥ 0 =⇒ 2(x2+y2+z2)−2(xy+xz+yz)≥ 0 =⇒ x2+y2+z2 ≥ xy+xz+yz.

Agora, usamos nosso problema mais simples para resolver o problema dado. Para isso, basta escrever

x = ab, y = ac, z = bc.

Assim, segue imediatamente a prova da desigualdade dada.

Exemplo-37:

É possível encontrar 2016 números inteiros positivos distintos, cada um deles sendo um quadradoperfeito, tal que a soma de todos eles seja também um quadrado perfeito?

Solução

Em vez de resolver a questão diretamente, é mais confortável pensar no problema seguinte, que é massimples e permite a solução do problema que queremos resolver:

Como veremos a resposta é sim.Se o problema fosse encontrar dois números inteiros positivos quadrados perfeitos cuja soma fosse umquadrado perfeito, usaríamos o teorema de Pitágoras:

32 +42 = 52.

Agora, vamos ver que esta situação mais simples nos revela uma saída para o problema mais complexo. Defato, multiplique cada lado da última igualdade por 52, obtendo

32.52 +42.52 = 52.52

e substitua 52 na primeira parcela do lado esquerdo por 32 +42. Assim, obtemos

32(32 +42)+42×52 = 52⇐⇒ 32.32 +32.42 +42.52 = 52.52.

Novamente, multiplicando cada lado da última igualdade por 52, obtemos

32.32.52 +32.42.52 +42.52.52 = 52.52.52

Page 38: NOTAS DE AULA - UFRGS

38 Capítulo 2. ESTRATÉGIAS

e usando na primeira parcela à esquerda a substituição de 52 por 32 +42, obtemos:

32.32.32 +32.32.42 +32.42.52 +42.52.52 = 52.52.52.

Continuando esse processo, encontraremos 2016 números inteiros positivos distintos, todos quadradosperfeitos, tal que a soma deles seja também um quadrado perfeito.

Exemplo-38:

Calcular a área lateral do tronco de cone de altura h e raios R e r, veja figura a seguir.

h

R

r

Solução

O tronco de cone é uma superfície tridimensional. Em vez de resolver a questão diretamente, é maisconfortável pensar num problema no plano. Assim, cortamos o tronco de cone ao longo de um segmento dereta, e colocamos sobre o plano, obtendo um trapézio, veja figura a seguir:

h

R

r

s

2πr

2πR

2πr

2πR

Agora, usamos que a área, S, do trapézio é igual a

S =(comprimento da base maior)+(comprimento da base menor)

2× (altura).

O comprimento da base maior é igual ao comprimento do círculo maior: 2πR.O comprimento da base menor é igual ao comprimento do círculo menor: 2πr.Resta calcular a altura do trapézio, que é igual ao comprimento do segmento da geratriz do tronco do cone.Seja g o comprimento do lado geratriz do tronco do cone, veja figura a seguir

h h g

R - rR

r

Page 39: NOTAS DE AULA - UFRGS

2.4 Começar pelo fácil torna fácil o difícil 39

Usando a figura acima e o Teorema de Pitágoras, temos

g2 = h2 +(R− r)2 =⇒ g =√

h2 +(R− r)2.

Portanto, a área lateral, S, do tronco de cone de altura h e raios R é igual a

S =2πR+2πr

2×√

h2 +(R− r)2 = π(R+ r)×√

h2 +(R− r)2.

Exemplo-39:

Se x, y, p são números reais positivos, com 0 < p < 1, prove a desigualdade

(x+ y)p < xp + yp

SoluçãoObserve que, se trocarmos x e y por tx e ty, com t positivo, respectivamente, a desigualdade continuavalendo. Ou seja, a desigualdade é invariante pela transformação que leva x em tx, y em ty. Istosignifica que, sem perda de generalidade, podemos supor que x+ y = 1. Mas, se x+ y = 1, com x, ypositivos, a desigualdade é óbvia, pois, neste caso, 0 < x < 1 e 0 < y < 1, implicando que x < xp ey < yp. Portanto a desigualdade é verdadeira.

Exemplo-40:

Resolva a seguinte equação:

(x2−4x+2014) · (x2−4x+2016)+20152 = 0

Solução

Em vez de resolver a questão diretamente, é mais confortável pensar numa alternativa que possa simplificara expressão do lado esquerdo da equação dada.

Para isso, vamos introduzir uma nova variável, com a qual possamos simplificar os dados do problema.Assim, seja

y = x2−4x−1.

Substituindo y na equação dada, obtemos:

(x2−4x+2014) · (x2−4x+2016)+20152 = 0⇐⇒ (y+2015) · (y−2015)+20152 = 0

⇐⇒ y2−20152 +20152 = 0⇐⇒ y2 = 0⇐⇒ y = 0.

Portanto,y = 0⇒ x2−4x−1 = 0⇒ x = 2±

√5.

Page 40: NOTAS DE AULA - UFRGS

40 Capítulo 2. ESTRATÉGIAS

2.5 Experimente, procure algo que seja invarianteMuita matemática foi feita por tentativa, errando, corrigindo, aperfeiçoando, avançando.As vezes pode ser vantajoso introduzir no problema algo novo, um auxílio extra, para que facilite você napercepção entre o que foi dado e o que foi pedido.

Exemplo-41:

Um jogo consiste de 9 botões luminosos (de cor verde ou vermelho) dispostos num quadrado damaneira seguinte:

1 2 3

4 5 6

7 8 9

Apertando um botão do bordo do quadrado, mudam de cor esse botão e todos seus vizinhos.Apertando o botão do centro, mudam de cor seus 8 vizinhos mas ele não.Os exemplos seguintes mostram com círculos pretos as luzes que mudam de cor ao serem pressionadoo botão que se indica.

∗∗∗∗ ∗

Botão 1

∗ ∗∗Botão 2

Botão 5

É possível (apertando sucessivamente alguns botões) acender todas as luzes com cor verde, seinicialmente estavam todas acesas com a cor vermelha? Justifique a resposta.

Solução

(Olimpíada de Matemática do Cone Sul) Inicialmente, observamos que, ao apertar um botão qualquer,mudam de cor um número par de botões, de acordo com a tabela seguinte.

Botão No Mudam de Cor1, 3, 7 ou 9 4 botões2, 4, 6 ou 8 6 botões

5 8 botões

Assim, a quantidade de botões que mudam da cor vermelho para a cor verde possui a mesma paridade que aquantidade de botões que mudam de verde para vermelho. Desta maneira, sempre haverá uma quantidadepar de botões verdes, de modo que nunca se poderá chegar a situação de se ter 9 botões verdes (em todo oquadrado), pois 9 é um número ímpar.De fato, seja an a quantidade de botões com a cor verde depois de n apertos sucessivos. Se v é aquantidade de botões que mudam de vermelho para verde e r é a quantidade de botões que mudam deverde para vermelho, no passo seguinte teremos

an+1 = an + v− r.

Mas, como (v− r) é um número par, segue que an+1 e an tem a mesma paridade. Como inicialmenteao = 0, implica que an é par para todo valor de n. Portanto, an 6= 0.

Page 41: NOTAS DE AULA - UFRGS

2.5 Experimente, procure algo que seja invariante 41

Exemplo-42:

Tem-se quarenta e três pedaços de palitos, cujos comprimentos são 1,2,3, · · · ,42,43 centímetros,respectivamente.Diga, justificando, se é possível formar um quadrado usando todos estes pedaços, sem quebrarqualquer um deles, nem sobrepor dois ou mais deles.E se em vez de um quadrado for um retângulo?

SoluçãoAqui é conveniente observar que

1+2+3+ · · ·+42+43 =43× (43+1)

2= 43×22 = 946.

Se fosse possível formar um quadrado, de lado medindo x, usando todos estes pedaços, sem quebrarqualquer um deles, nem sobrepor dois ou mais deles, teríamos necessariamente que o perímetro doquadrado seria igual a 946. Ou seja, 4x = 946, o que é impossível, pois x tem de ser um número inteiro eo número 946 não é divisível por 4.Se em vez de um quadrado for um retângulo, com lados medindo a e b centímetros, teríamos o perímetrodo retângulo como sendo igual a 2a+2b = 946, ou seja a+b = 473. Neste caso, a resposta é sim. Se osquatro lados do retângulo medem a,a,b,b, poderíamos tomar:

a = 1+2; e a = 3, e

b = (4+43)+(6+41)+(8+39)+(10+37)+ · · ·+(19+29)+(20+27)+(22+25)

e

b = (5+42)+(7+40)+(9+38)+(11+36)+ · · ·+(19+28)+(21+26)+(23+24).

Exemplo-43:

Um tabuleiro 10×10 pode ser coberto por 25 dominós de dimensões 1×4?

SoluçãoAqui o invariante mais óbvio é a área, que infelizmente não ajuda na solução do problema. Por outro lado,podemos escrever um número em cada um dos 100 quadrados unitários do tabuleiro com a propriedadeque, não importa como nós colocamos um dos dominós cobrindo 4 quadrados, a soma dos números emquatro quadrados coberto seja igual a zero; no entanto, a soma de todos os 100 números em todo otabuleiro não é zero. Então, obviamente, a cobertura é impossível.Como associar os números aos quadrados unitários do tabuleiro do modo com prevemos acima?Defina os valores an, para n = 1,2,3, · · · ,9,10, da seguinte maneira:

a1 = 1 a2 = 1 a3 = 1 a4 =−3 a5 = 1 a6 = 1 a7 = 1 a8 =−3 a9 = 1 a10 = 1

Nesta sequência, qualquer bloco consecutivo de quatro termos tem soma igual a 0, mas a soma de todos osdez não é 0. Agora, escrevemos no quadrado unitário do tabuleiro que está na linha m e coluna n o númeroam×an, que nos sugere uma pintura para o tabuleiro usando três cores distintas, veja figura a seguir.

Page 42: NOTAS DE AULA - UFRGS

42 Capítulo 2. ESTRATÉGIAS

1

1

-3

1

1

1

-3

1

1

1

1

1

-3

1

1

1

-3

1

1

1

1

1

-3

1

1

1

-3

1

1

1

-3

-3

9

-3

-3

-3

9

-3

-3

-3

1

1

-3

1

1

1

-3

1

1

1

1

1

-3

1

1

1

-3

1

1

1

1

1

-3

1

1

1

-3

1

1

1

-3

-3

9

-3

-3

-3

9

-3

-3

-3

1

1

-3

1

1

1

-3

1

1

1

1

1

-3

1

1

1

-3

1

1

1

Exemplo-44:

Três máquinas caça-niqueis imprimem pares de inteiros positivos em cartões. As máquinas caça-níqueis trabalham da seguinte maneira. A primeira máquina caça-níquel depois de ler um cartão(a,b) imprime um novo cartão (a+ 1,b+ 1); a segunda máquina caça níquel depois de ler umcartão (a,b) imprime um novo cartão ( a

2 ,b2 ). Esta máquina só funciona se os números a e b

forem pares. A terceira máquina caça-níquel, depois de ler dois cartões (a,b) e (b,c) imprime umnovo cartão (a,c). Além disso, as máquina caça-níquel devolvem todos os cartões lidos.É possível começar com um cartão (4,18) e chegar num cartão (1,100)?

Solução(AUMO - 1978) A resposta é não. Para qualquer cartão (a,b) chame D(a,b) a diferença a−b.Seja S a coleção de todos os cartões obtidos a partir do cartão (4,18) por meio de operaçõesconsecutivas das máquinas caça-níquel.Vamos mostrar que D(a,b) é divisível por 7, para cada (a,b) ∈ S. Isto é, o resto da divisão de D(a,b)por 7 é um invariante.Para mostrar isso, vamos usar indução sobre o número de sucessivos usos das máquinas.Observe que D(4,18) = 4−18 =−14. Logo, o resto da divisão de D(4,18) por 7 é zero:−14 = (−2)×7+0. Vamos supor que depois do n−ésimo uso das máquinas tenhamos um cartão (a,b),com o resto da divisão de D(a,b) por 7 igual a zero.Vamos considerar o (n+1)−ésimo uso das máquinas caça-niquel. Existem três casos possíveis:• Usando a primeira máquina, depois de colocar o cartão (a,b), obtemos o cartão (a+1,b+1).

Neste caso, D(a+1,b+1) = (a+1)− (b+1) = a−b, que, por hipótese de indução, é divisível por7.• Usando a segunda máquina, depois de colocar o cartão (a,b), obtemos o cartão ( a

2 ,b2 ).

Neste caso, D( a2 ,

b2 ) =

a2 −

b2 ) =

12 (a−b), que, por hipótese de indução, é divisível por 7.

• Usando a terceira máquina, depois de colocar os cartões (a,b) e (c,d), obtemos o cartão (a,c).Neste caso, D(a,c) = a− c = a−b+b− c = (a−b)+(b− c), que, por hipótese de indução, édivisível por 7, por ser soma de dois números múltiplos de 7.

Assim, em todos os três casos, o resto da divisão de D(a,b) por 7 no novo cartão é nulo.O mesmo é verdade para o caso do cartão obtido a partir de (4,18) nas três máquinas caça-niquel. Mas,D(1,100) = 1−100 =−99 = (−15)×7+6. Portanto, D(1,100) deixa resto 6 na divisão por 7,consequentemente não é possivel obter o cartão (1,100) a partir do cartão (4,18).

Page 43: NOTAS DE AULA - UFRGS

2.6 Faça um desenho e, dependendo da situação, pinte às cores. 43

2.6 Faça um desenho e, dependendo da situação, pinte às cores.

Muitos de nós pensamos melhor com um desenho, esquema, imagem, do que com palavras. Diz o ditadopopular: uma imagem vale mais do que mil palavras.Logo, sempre que puder, faça um esquema auxiliar, um desenho, pinte às cores. Essas atitudes, quasesempre revelam caminhos surpreendentemente elegantes e fáceis de comprovar.

Exemplo-45:

Considere um tabuleiro de xadrez 8×8 e 32 dominós de dimensão 2×1. Os dominós podem serarranjados sobre o tabuleiro de modo a cobri-lo inteiramente, cada dominó cobrindo perfeitamenteduas casas e não havendo sobreposição de dominós. Do tabuleiro, retiram-se duas casas, situadasnos cantos do tabuleiro de uma mesma diagonal, veja Figura a seguir.

Diga, justificando, se 31 dominós cobrem completamente o tabuleiro reduzido.

Solução

A resposta é não.

A ideia aqui é pintar o tabuleiros de modo que as casas fiquem alternadamente pretas e brancas, veja figuraa seguir.

Observe que os dois quadrados situados nos cantos do tabuleiro possuem a mesma cor e que cada dominópor inteiro cobre sempre uma casa de cada cor, independente de como você o coloca sobre o tabuleiro nahorizontal ou vertical. Assim, se a cobertura fosse possível deveríamos ter no tabuleiro a mesma quantidadede casas de cada cor, o que não ocorre com a retirada de duas casas de mesma cor.Portanto, 31 dominós não cobrirão completamente o tabuleiro reduzido.

Page 44: NOTAS DE AULA - UFRGS

44 Capítulo 2. ESTRATÉGIAS

Exemplo-46:

Escreve-se um número em cada uma das 16 casas de um tabuleiro 4×4 . Para qualquer casa,a soma dos números escritos nas casas que tem um lado comum com ela é igual a 1. Determine asoma dos 16 números escritos nas casas do tabuleiro.

Solução (Torneio das Cidades - 2000) A resposta é 6.

Pinte as casas do tabuleiro alternadamente branca e marron. Cada casa branca é vizinha, com um ladocomum, de uma das casas marrons assinaladas, veja figura a seguir.

X

X

X

Observe que, pela hipótese do problema, podemos concluir que a soma dos números nas casas brancas éigual a 3. Resta mostrar que a soma dos números nas casa marrons é igual a 3. Para isso, preencha ocentro do tabuleiro com 0 e as demais casas como na figura a seguir.

z

y

x

w

w

0

0

z y

0

0

x

x

w

z

y

Como x+ z = 1 e y+w = 1, segue que a soma dos números escritos nas casas do tabuleiro é igual a

3(x+ z)+3(y+w) = 3 ·1+3 ·1 = 3+3 = 6.

Exemplo-47:

Coloca-se uma ficha em cada um dos seis círculos da figura a seguir.

O jogador pode escolher quaisquer duas fichas e movimentá-las em direções opostas pelo mesmonúmero de passos. O objetivo é colocar todas as fichas num mesmo círculo, formando uma pilha.(a) O jogador consegue alcançar seu objetivo? Se ele consegue, descreva um método. Se não, proveque isto é impossível.(b) E se, em vez de seis, fossem 10 círculos?

Page 45: NOTAS DE AULA - UFRGS

2.6 Faça um desenho e, dependendo da situação, pinte às cores. 45

Solução

(a) É impossível.Pintemos os círculos alternadamente de branco e preto, veja figura a seguir.

Seja S a quantidade de fichas nos círculos brancos em determinado instante. Inicialmente, S = 3. Observeque, após cada movimento do jogador, ou S aumenta de 2, ou S diminui de 2 ou S não se altera. Ora,isto significa que S permanece um número ímpar durante todo o jogo. Por outro lado, se todas as fichasficarem num mesmo círculo, formando uma pilha com seis fichas, teremos: ou S = 6 ou S = 0, que sãodois números pares. Portanto, é impossível colocar todas as fichas num mesmo círculo.(b) Neste caso, é possível.Observe que n = 10 = 2 ·5. Ou seja, n = 2k, com k um número natural ímpar. Vamos mostrar que, nocaso em que n = 2k, com k um número natural ímpar, é possível colocar todas as fichas num mesmocírculoNumere os círculos consecutivamente, no sentido horário, por: 1, 2, 3, 4, . . . , n.Suponha que, em determinado instante, tenhamos ai fichas no círculo numerado com i e seja S aexpressão da soma das fichas nos círculos nesse instante. Isto é:

S = 1 ·a1 +2 ·a2 +3 ·a3 + . . .+n ·an

Observe que, inicialmente ai = 1, para todo i = 1, 2, 3, . . . , n. Assim, inicialmente temos:

S = 1 ·1+2 ·1+3 ·1+ . . .+n ·1 =n(n+1)

2.

Após cada movimento, acontece o seguinte com o valor da expressão da soma das fichas em cada um dos ncírculos: ela não se altera ou ela se altera por n. Por exemplo, para n = 6 = 2×3. Inicialmente temos:

S = 1 ·1+2 ·1+3 ·1+4 ·1+5 ·1+6 ·1 =6(6+1)

2= 21.

Se fizermos o movimento de colocar a ficha do círculo de número 2 para o círculo de posição 4 e passar aficha do círculo da posição 6 para o círculo da posição 4, a expressão S, que inicialmente é 21, torna-se:

S = 1 ·1+2 ·0+3 ·1+4 ·3+5 ·1+6 ·0 = 1+3+12+5 = 21.

Por outro lado, fizermos o movimento de colocar a ficha do círculo de número 2 para o círculo de posição4 e passar a ficha do círculo da posição 1 para o círculo da posição 5, a expressão S, que inicialmente é21, torna-se:

S = 1 ·0+2 ·0+3 ·1+4 ·2+5 ·2+6 ·1 = 3+8+10+6 = 27.

Assim, após vários movimentos a soma S torna-se

S =n(n+1)

2+q ·n, onde q ∈ Z.

Agora, quando todas as fichas encontram-se no círculo de número k, temos

S = 1 ·0+2 ·0+3 ·0+ . . .+(k−1) ·0+ k ·n+(k+1) ·0+ . . .+n ·0 = k ·n.

Page 46: NOTAS DE AULA - UFRGS

46 Capítulo 2. ESTRATÉGIAS

Assim, temos

n(n+1)2

+q ·n = k ·n ⇐⇒ n+12

+q = k ⇐⇒ n+1 = 2k−2q ⇐⇒ n = 2(k−q)−1.

Portanto, n tem de ser um número ímpar, que é uma condição necessária para que o jogador possaalcançar seu objetivo. É fácil mostrar que esta condição é também suficiente. De fato, podemos destacarqualquer um dos círculo, digamos o círculo numerado com 1. Como n é um número ímpar, os restantesn−1 círculos podem ser agrupados em pares. Cada par é formado por dois círculo escolhidos de modo queambos estejam à mesma distância do círculo 1, sendo que colocados em direções opostas. Isto é, os paressão formados pelos círculos numerados por (i,n− i+2), para i = 2,3, . . . ,n. As fichas dos dois círculosque formam um par podem ser levadas para o círculo numerado com 1 em um movimento. Portanto, nestecaso, o jogador pode colocar todas as fichas num mesmo círculo, formando uma pilha.

Exemplo-48:

O rei, peça do jogo de xadrez, se movimenta uma única casa por vez em todas as direções. Pode orei, sem fazer movimentos diagonais, começando da casa mais abaixo, à esquerda, ir até a casa maisacima à direita, visitando cada uma das casas restantes do tabuleiro exatamente uma única vez?

Solução

Depois de ler o problema (se preciso for, mais de uma vez, talvez até em voz alta), de forma a saberexatamente quais as quantidades dadas, quais as condições dadas (hipóteses) e o que se pede, vamos traçarum plano para a solução.

A resposta é não.Pinte as casas do tabuleiro alternadamente pretas e brancas. Agora, basta observar que o rei, a cadamovimento, sem fazer trajetos diagonais, vai de uma casa de uma cor para a casa de cor oposta. Como o reitem que fazer 63 desses movimentos para tingir a casa mais acima à direita, o último movimento terá quedeixá-lo em uma casa da cor oposta a cor da primeira casa, pois 63 é um número ímpar . Mas, a primeiracasa e a última casa possuem a mesma cor, o que é uma contradição. Portanto, o rei não pode, sem fazermovimentos diagonais, começando do quadrado mais abaixo, à esquerda, ir até o quadrado mais acima àdireita, visitando cada uma das casas restantes do tabuleiro exatamente uma única vez.

Exemplo-49:

Numa sala quadrada de dimensões 13× 13 retira-se o quadrado unitário central. É possívelladrilhar o restante da sala usando ladrilhos retangulares de dimensões 1×4 ou 4×1?

Solução

Depois de ler o problema (se preciso for, mais de uma vez, talvez até em voz alta), de forma a saberexatamente quais as quantidades dadas, quais as condições dadas (hipóteses) e o que se pede, vamos traçarum plano para a solução.

Não é possível.Pinte os quadrados do assoalho de preto e branco no padrão seguinte. Na primeira linha acima, pinte depreto os dois quadrados mais à esquerda, pinte de branco os dois seguintes (da esquerda para direita), osdois próximos pinte de preto, os dois próximos de branco, e assim por diante, de modo que na extremidadedireita fique um único quadrado preto. Na segunda linha, pinte a linha de forma alternada com a pintura daprimeira linha: dois quadrados brancos, dois quadrados pretos e assim por diante. Numerando as linhas de1 a 13, temos que todas as linhas ímpar são pintadas de forma idêntica que a pintura da primeira linha etodas as linhas numeradas com números pares são pintadas de forma idêntica que a pintura da linha denúmero 2, veja figura a seguir.

Page 47: NOTAS DE AULA - UFRGS

2.6 Faça um desenho e, dependendo da situação, pinte às cores. 47

X

Observe que existem mais quadrados unitários pretos do que quadrados brancos. Por outro lado, cadaladrilho 4×1, não importa como seja colocado, cobre precisamente dois quadrados unitários pretos e doisbrancos. Assim, se um ladrilhamento deixar um único quadrado descoberto, este quadrado tem de ser preto.Mas, o quadrado central é branco. Portanto, tal ladrilhamento é impossível.

Exemplo-50:

Uma peça L é formada a partir de quadrados 2× 2, dos quais retiramos um dos quatrocantos, veja Figura A a seguir. Diga, justificando, se é possível cobrir um tabuleiro 5× 7 ,Figura B a seguir, usando peças L′s, de modo que a cobertura, em várias camadas, não cruzeas fronteiras do tabuleiro e de maneira tal que cada casa seja coberta pelo mesmo número de peças L.

Figura A- Peça L Figura B - Tabuleiro 5×7

Solução

(Rússia - 1996). A resposta é não.Pintamos as casas do tabuleiro dado 5×7 alternadamente de preto e branco, de modo que as casas nosquatro cantos sejam pretas. Em cada casa preta preta escrevemos o número −2, e em cada casa brancaescrevemos o número 1. Observe que a soma dos números em todas as casas cobertas por cada peça L énão negativa, e consequentemente se cobrimos o tabuleiro em k níveis, a soma dos números das casascobertas por aquelas peças L é não negativa. Mas, se o número é S e s é a soma dos números sobre otabuleiro, temos que

S = ks = k(−2×12+23×1) =−k < 0,

que é uma contradição.

Page 48: NOTAS DE AULA - UFRGS

48 Capítulo 2. ESTRATÉGIAS

Exemplo-51:

Em cada uma das 64 casas de um tabuleiro de xadrez 8×8 há um grão de açúcar. Uma formigacomeça em uma casa do canto do tabuleiro, come o açúcar, e se move para uma casa adjacente,movimentando-se em direção horizontal ou vertical (mas nunca na diagonal). Continua deste modoaté acabar com todo o açúcar, e sem passar duas vezes por uma mesma casa. É possível que otrajeto da formiga termine no canto do tabuleiro diagonalmente oposto ao inicial?

Solução

Vamos demonstrar que não existe trajetória que satisfaça as condições exigidas.Para isso, é útil pensar as casas do tabuleiro pintadas alternadamente de branco e preto. Agora, observeque em cada movimento unitário na direção horizontal ou vertical leva a formiga de uma casa para outra decor distinta da imediatamente anterior. Como o tabuleiro possui 8×8 = 64 casas, começando emqualquer uma delas são necessários 63 movimentos para percorrer todas. Mas, é fácil ver que depois1, 3, 5 ou qualquer quantidade impar de movimentos estaremos em uma casa de cor diferente da inicial.Portanto, a resposta ao problema é negativa, pois a casa do canto diagonalmente oposto é de mesma cor.

Exemplo-52:

Seja P um ponto na região limitada pelo triângulo ABC. Traçam-se por P as paralelas aos ladosdo triângulo, que subdividem o triângulo em três triângulos e três paralelogramos. Se as áreas dostrês triângulos da subdivisão são, em alguma ordem, 9, 16 e 25, encontrar a área do triânguloABC.

Solução

Para ajudar na solução, fazemos a seguir um desenho ilustrativo da situação.

A B

C

P•

Para resolver o problema usaremos o seguinte fato: “Se dois triângulos são semelhantes então a razão entresuas áreas é igual a razão entre lados homólogos.”Agora, observe que os três triângulos da sub-divisão são semelhantes ao triângulo ABC, pois possuemlados respectivamente paralelos.Sejam S a área do triângulo ABC e x, y, z os comprimentos dos lados paralelos ao lado AB dostriângulos de área 9, 16, 25, respectivamente.As retas por P, paralelas aos lados BC e AC dividem o lado AB em três segmentos de comprimentosx, y, z (em alguma ordem). Assim, o comprimento do lado AB satisfaz:

AB = x+ y+ z.

Logo, temos ( xx+ y+ z

)2=

9S,( y

x+ y+ z

)2=

16S,( z

x+ y+ z

)2=

25S,

que é equivalente ax

x+ y+ z=

3√S,

yx+ y+ z

=4√S,

zx+ y+ z

=5√S.

Page 49: NOTAS DE AULA - UFRGS

2.6 Faça um desenho e, dependendo da situação, pinte às cores. 49

Somando membro a membro as três igualdades acima, temos :

xx+ y+ z

+y

x+ y+ z+

zx+ y+ z

=3√S+

4√S+

5√S=

12√S⇔ 1 =

12√S⇔ S = 144.

Page 50: NOTAS DE AULA - UFRGS

50 Capítulo 2. ESTRATÉGIAS

2.7 Modifique o enunciado, para ver se lhe ocorre um caminho possívelQuando o problema parecer difícil demais, modifique o enunciado, transforme o problema em outro, istopode levar a um caminho que lhe mostre a saída.

Exemplo-53:

Encontre todos os pares de inteiros positvos (x,y) para os quais temos

3x+

1y=

12

e√

xy≥ 3√

6.

Solução

Depois de ler o problema (se preciso for, mais de uma vez, talvez até em voz alta), de forma a saberexatamente quais as quantidades dadas, quais as condições dadas (hipóteses) e o que se pede, vamos traçarum plano para a solução.

Em vez de olhar para3x+

1y=

12

, vamos pensar numa equação equivalente a ela:

3x+

1y=

12⇔ 2x+6y = xy ⇔ xy−2x−6y = 0 ⇔ xy−2x−6y+12 = 0+12 ⇔ (x−6) · (y−2) = 12.

Assim, os possíveis pares de inteiros positivos satisfazendo as condições acima são os elementos doconjunto:

(18,3), (12,4), (10,5), (9,6), (8,8), (7,14).

Por outro lado, temos que os elemntos de cada par tem de satisfazer:

√xy≥ 3

√6 ⇔ xy≥ 32×6 = 54.

Portanto, os pares de inteiros positvos (x,y) que queremos são: (18,3), (9,6), (8,8), (7,14).

Exemplo-54:

Calcule o valor da expressão E = (tan15◦).(tan30◦).(tan45◦).(tan60◦).(tan75o◦).

Solução

A ideia aqui é óbservar que se x+ y = 90◦, então

tan (x+ y) =tan x+ tan y

1− tan x. tan y= tan 90◦ = ∞.

Ou seja, 1− tan x◦. tan y◦ = 0, que é o mesmo que tan x◦. tan y◦ = 1, ou ainda

tan x◦ =1

tan y◦=

1tan (90o− x◦)

= cot (90o− x◦)

Logo, temos quetan75◦ = cot (90◦−75◦) = cot 15◦;

tan60◦ = cot (90◦−60◦) = cot 30◦.

Portanto,E = (tan15◦).(tan30◦).(tan45◦).(tan60◦).(tan75◦) =

= (tan15◦).(cot 15◦).(tan30◦).(cot 30◦).(tan45◦) = 1.1.1 = 1.

Page 51: NOTAS DE AULA - UFRGS

2.7 Modifique o enunciado, para ver se lhe ocorre um caminho possível 51

Exemplo-55:

Escreve-se em cada uma das seis faces de um cubo um número inteiro positivo. Para cada vértice docubo, calcula-se o produto dos números nas três faces adjacentes a ele. A soma desses produtos éigual a 1001Qual é a soma dos seis números escritos nas faces do cubo?

Solução

Depois de ler o problema (se preciso for, mais de uma vez, talvez até em voz alta), de forma a saberexatamente quais as quantidades dadas, quais as condições dadas (hipóteses) e o que se pede, vamos traçarum plano para a solução.

Em vez de tomar aleatoriamente os seis números escritos nas faces do cubo, vamos considerar os númerosdois a dois, escritos nas faces opostas. Assim, sejam a1,a2,b1,b2,c1,c2 os números escritos nas faces doscubos de tal maneira que a1,a2 estão escritos em faces opostas, b1,b2 números ecritos em faces opostas ec1,c2 números escritos em faces opostas. Agora, a soma dos oito produtos:

a1.a2 +a1.b1 +a1.b2 +a1.c1 + · · ·+ c1.c2 = 1001.

Por outro lado,

a1.a2 +a1.b1 +a1.b2 +a1.c1 + · · ·+ c1.c2 = (a1 +a2).(b1 +b2).(c1 + c2) =

= 1001 = 7.11.13

. Portanto, a soma dos oito números escritos nas faces do cubo é igual a:

(a1 +a2)+(b1 +b2)+(c1 + c2) = 7+11+13 = 31.

Exemplo-56:

Pintam-se ou de vermelho ou de verde 2001 pontos distintos sobre um círculo. Em cada etapa,todos os pontos são simultaneamente pintados novamente da maneira seguinte:se dois pontos são vizinhos imediatos de um ponto P e possuem a mesma cor que P, então a cor deP não se altera, caso contrário P muda de cor.Começando com a primeira pintura F1, obtemos as pinturas F2, f3, · · · , depois de aplicar váriasetapas de pinturas.Prove que existe um número n0 ≤ 1000 tal que Fn0 = Fn0+2. Isto é verdade se o número 1000 forsubstituído pelo número 999?

Solução

Depois de ler o problema (se preciso for, mais de uma vez, talvez até em voz alta), de forma a saberexatamente quais as quantidades dadas, quais as condições dadas (hipóteses) e o que se pede, vamos traçarum plano para a solução.

(BW 2001) A resposta é não.Vamos denotar os pontos sobre o círculo por 1,2,3, · · · ,2001, de tal modo que os pontos i, j sejam vizinhosse |i− j|= 1 ou {i, j}= {1,2001}.Vamos dizer que k pontos formam um segmento monocromático de comprimento k se os pontos sãoconsecutivos sobre o círculo e se eles possuem a mesma cor.Para uma pintura F, chamamos de d(F) o comprimento máximo de um segmento monocromático.Observe que d(F)> 1, para todo n, pois 2001 é ímpar. Se d(F) = 2001, então todos os pontospossuem a mesma cor, portanto, F1 = F2 = F3 = · · · e, neste caso, podemos escolher n0 = 1.Seja 1 < d(F1)< 2001. Valem as seguintes propriedades:(i) Se 3 < d(Fn)< 2001, então d(Fn+1) = d(Fn)−2;

Page 52: NOTAS DE AULA - UFRGS

52 Capítulo 2. ESTRATÉGIAS

(ii)Se d(Fn) = 3, então d( fn+1) = 2;(iii) Se d(Fn) = 2, então d(Fn+1 = d(Fn e fn+2 = fn;Se valem as três propriedades acima, segue de (i) e (ii) que d(F1000)≤ 2 e por (iii) temos queF1000 = F1002. Além disso, se F1 é a pintura onde 1 é pintado de vermelho e todos os outros pontos sãopintados de verde, então d(F1) = 2000 e então d(F1)> d(F2)> · · ·> d(F1000) = 2, o que mostra quepara todo n < 1000, temos Fn 6= Fn+2 e então 1000 não pode ser substituído por 999.Agora, vamos provar as três propriedades acima.Seja (i+1, i+2, . . . , i+ k) o maior segmento monocromático para Fn, considerando os pontos módulo2001. Então (i+2, i+3, . . . , i+ k−1) é um segmento monocromático para Fn+1 e segue qued(Fn+1)≥ d(Fn−2. Além disso, se (i+1, i+2, . . . , i+ k) é o maior segmento monocromático para Fn+1,com k ≥ 3, então (i, i+1, . . . , i+ k+1) é um segmento monocromático para Fn. Disso, e do fato deFn+1 > 1, seguem (i) e (ii).Para provar (iii) observe que se d(Fn)≤ 2, então Fn+1 é obtido de Fn mudando as cores de todos ospontos, o que conclui a prova.

Page 53: NOTAS DE AULA - UFRGS

2.8 Explore a simetria 53

2.8 Explore a simetria

Muitos problemas são resolvidos quando se explora a simetria que o problema apresenta, de forma explícitaou mesmo mascarada.

Exemplo-57:

O professor de Matemática de Joãzinho apresenta um tabuleiro 6×6, veja Figura a seguir,

e propõe o seguinte desafio: Joãozinho tem de pintar de preto doze casas do tabuleiro, de modoque em cada linha e em cada coluna apareçam somente duas casas pintadas e, ao longo de umadiagonal, não mais do que duas.Joãozinho vai conseguir resolver o desafio? Se sim, mostre como. Se não, explique porque.

Solução

Depois de ler o problema (se preciso for, mais de uma vez, talvez até em voz alta), de forma a saberexatamente quais as quantidades dadas, quais as condições dadas (hipóteses) e o que se pede, vamos traçarum plano para a solução.

Joãozinho vai resolver o desafio. A resposta não é única. A estratégia de Joãozinho consiste em dividir otabuleiro em quatro sub-tabuleiros 3×3, trabalhar num desses sub-tabuleiros e, em seguida, usar reflexãoem torno das retas que dividem o tabuleiro e torno de seu centro. A seguir, apresentamos duas soluções.

Page 54: NOTAS DE AULA - UFRGS

54 Capítulo 2. ESTRATÉGIAS

Exemplo-58:

Inscreve-se um quadrado num círculo, que por sua vez está inscrito noutro quadrado, veja figura aseguir.

Qual é a razão das áreas dos dois quadrados?

Solução

Podemos resolver o problema usando a noção de simetria. Se você gira o quadrado menor por um ângulo de45◦, seus vértices irão coincidir com os pontos de tangência do círculo e do quadrado maior, veja figura aseguir.

Portanto, é fácil ver, que o quadrado menor possui a metade da área do quadrado maior.

Exemplo-59:

Eve e Odette disputam um jogo num tabuleiro 3×3, com peças brancas e peças pretas. As regrassão as seguintes:I - Elas jogam alternadamente;II - Uma jogada consiste em colocar uma peça numa casa do tabuleiro ainda não ocupadaIII - Na sua vez de jogar, uma jogadora pode escolher ou uma peça branca ou uma peça preta e nãoprecisa usar sempre a mesma cor;IV - Quando o tabuleiro está totalmente preenchido pelas peças, Eves obtém um ponto para todalinha, coluna ou diagonal que possuir um número par de peças pretas, e Odette obtém um pontopara toda linha, coluna ou diagonal que possuir um número ímpar de peças pretas;V - A jogadora que obtiver no mínimo cinco dos oito pontos VENCE.Descreva uma estratégia vencedora para a garota que começa o jogo.

Solução

(CMO 1978)Vamos supor que Eve começa o jogo.Eve coloca uma peça preta na casa central do tabuleiro, ela existe pois no tabuleiro temos 9 casas. A partirdaí, sempre que Odette jogar, Eve joga com uma peça da cor oposta a de Odette na casa simétrica, emrelação a casa central, daquela que Odette jogou. Por este procedimento, Eve obtém um ponto para todalinha, coluna e diagonal que passem pelo centro. Além disso, pelo menos uma das linhas ou colunas quepassam pelo centro tem 2 peças pretas, o que implica que Eves vence o jogo.Chamando de B uma peça branca e P uma peça preta, no final do jogo o tabuleiro terá a seguinteaparência:

P 1 BP

P 2 B

Page 55: NOTAS DE AULA - UFRGS

2.8 Explore a simetria 55

Exatamente uma das casas 1 e 2 possui uma peça preta, e a linha dela possui um número par de peçaspretas.

Page 56: NOTAS DE AULA - UFRGS

56 Capítulo 2. ESTRATÉGIAS

2.9 Redução ao Absurdo.Um raciocínio muito empregado na resolução de problemas é aquele que comumente se chama de métodopor redução ao absurdo ou método indireto.Como é que funciona?Se você pretende mostrar que uma afirmação A implica numa afirmação B, comece supondo que isto nãoacontece. Ou seja, que A não implica em B. Fazendo deduções e raciocínios corretos, você chegará àconclusão de que uma afirmação, que você sabe que é correta, não se realiza. Ou seja, você chegará a umacontradição. Essa contradição provém do fato de termos assumido que A não implica em B. Portanto, Aimplica em B.

Exemplo-60:

Demonstrar que o número√

2 é irracional.

Solução

Depois de ler o problema (se preciso for, mais de uma vez, talvez até em voz alta), de forma a saberexatamente quais as quantidades dadas, quais as condições dadas (hipóteses) e o que se pede, vamos traçarum plano para a solução.

Vamos provar por absurdo. Suponha o contrário, isto é,√

2 não é um número irracional. Isto significadizer que √

2 =mn, onde m,n ∈ Z, com n 6= 0.

Sem perda de generalidade, vamos supor que os números m e n sejam relativamente primos. Ou seja,MDC(m,n) = 1. Elevando ao quadrado ambos os membros de

√2 = m

n , obtemos 2 = m2

n2 , que é o mesmoque dizer que

m2 = 2n2 (∗).Assim, m2 é um número par e, logo, m é par. Desse modo, m = 2k, onde k ∈ Z. Substituindo esse valorde m em (∗), obtemos que n2 é par, o que implica que n é par. Assim, concluímos que m é par e n épar. Uma contradição, pois tínhamos suposto MDC(m,n).= 1. Portanto, o número

√2 é irracional.

Exemplo-61:

Se a, b, c são números inteiros ímpares, prove que a equação quadrática

ax2 +bx+ c = 0 (∗)

não possui um número racional como solução.

Solução

Depois de ler o problema (se preciso for, mais de uma vez, talvez até em voz alta), de forma a saberexatamente quais as quantidades dadas, quais as condições dadas (hipóteses) e o que se pede, vamos traçarum plano para a solução.

Vamos provar por absurdo. Suponha o contrário, isto é, existe um número racional pq com MDC(p,q) = 1,

que seja solução da equação dada (∗). Assim, temos que

a( p

q

)2+b( p

q

)+ c = 0 ⇒ ap2 +bpq+ cq2 = 0. (∗∗)

Agora basta analisar a expressão (**) considerando a paridade de p e q:• Se p e q são ambos números ímpares.

Neste caso, a expressão ap2 +bpq+ cq2 é um número ímpar, pois a, b, c são números inteirosímpares. Portanto, ap2 +bpq+ cq2 6= 0, pois 0 é um número par.

• Se p é um número par e q é um número ímpar.Neste caso, temos que ap2 +bpq é um número par, enquanto cq2 é um número ímpar. Logo,ap2 +bpq+ cq2 é um número ímpar. Portanto, não pode ser zero.

Page 57: NOTAS DE AULA - UFRGS

2.9 Redução ao Absurdo. 57

• Se p é um número ímpar e q é um número par.Neste caso, ap2 é um número ímpar, enquanto bpq+ cq2 é par. Logo, ap2 +bpq+ cq2 é umnúmero ímpar. Portanto, não pode ser zero.

(Observe que que os números p,q não podem ser ambos pares, pois MDC(p,q) = 1). Assim, se a equaçãopossui uma solução racional, chegaremos a uma contradição. Portanto, a equação dada não possui umnúmero racional como solução.

Exemplo-62:

Escrevem-se os números 1, 2, 3, . . . , 49 nas casas de um tabuleiro 7×7, sendo um número emcada casa e calcula-se a soma dos números em cada linha e cada coluna. Algumas dessas 14 somassão ímpares enquanto as outras são pares. Seja A a soma de todas as somas ímpares e B a somade todas as somas pares.É possível colocar os números nas casas do tabuleiro de modo que A = B?

Solução

Depois de ler o problema (se preciso for, mais de uma vez, talvez até em voz alta), de forma a saberexatamente quais as quantidades dadas, quais as condições dadas (hipóteses) e o que se pede, vamos traçarum plano para a solução.

A resposta é não.Se isto fosse possível, teríamos 2× (1+2+3+4+ . . .+49) = A+B = 2B. Mas, B é um número par,pois é a soma de números pares. Por outro lado, 1+2+3+4+ · · ·+49 = 25×49, que é um númeroímpar. Contradição. Portanto é impossível colocar os números nas casas do tabuleiro de modo que A = B.

Exemplo-63:

Temos 2000 maçãs em várias bolsas. Podemos remover bolsas e/ou remover maçãs das bolsas.Prove que é possível ter um número igual de maçãs em cada uma das bolsas restantes, sendo o totalde maçãs não menor do que 100.

Solução

Depois de ler o problema (se preciso for, mais de uma vez, talvez até em voz alta), de forma a saberexatamente quais as quantidades dadas, quais as condições dadas (hipóteses) e o que se pede, vamos traçarum plano para a solução.

(TT - 1984) Suponha o contrário. Assim, o número total de bolsas restantes não é mais do que 99, casocontrário, poderíamos deixar 1 maçã em cada uma das 100 bolsas e remover as restantes. Além disso, ototal de bolsas com no mínimo duas maçãs não é mais do que 49, o total de bolsas com no mínimo trêsmaçãs não é mais do que 33, etc. Portanto, o total de maçãs não é maior do que 99+49+33+ . . ..Existem 94 termos depois dos cinco primeiros termos , cada um deles sendo menor do que ou igual a 16.Agora

99+49+33+24+19+94×16 = 1728 < 2000.

Contradição. Portanto, é possível ter um número igual de maçãs em cada uma das bolsas restantes, sendo ototal de maçãs não menor do que 100.

Exemplo-64:

Prove que não existem números inteiros positivos m e n para os quais

m2−n2 = 1. (∗)

Solução

Depois de ler o problema (se preciso for, mais de uma vez, talvez até em voz alta), de forma a saberexatamente quais as quantidades dadas, quais as condições dadas (hipóteses) e o que se pede, vamos traçarum plano para a solução.

Page 58: NOTAS DE AULA - UFRGS

58 Capítulo 2. ESTRATÉGIAS

Suponha o contrário. Isto é, suponha que existam números inteiros positivos m e n para os quaism2−n2 = 1. Assim, temos

m2−n2 = 1 ⇐⇒ (m+n) · (m−n) = 1 (∗∗)

Como m e n são inteiros positivos, segue que m+n e m−n são números inteiros. Logo, de (∗∗)concluímos que• m+n = 1 e m−n = 1 (∗∗∗)

ou• m+n =−1 e m−n =−1 (∗∗∗∗)

Agora, somando membro a membro as duas equações de (∗∗∗), obtemos 2m = 2 ⇒ m = 1 ⇒ n = 0.Contradição, pois n é um número inteiro positivo.De módulo análogo, somando membro a membro as duas equações de (∗∗∗∗), obtemos 2m = 0 ⇒ m = 0.Contradição, pois m é um número inteiro positivo.Portanto, não existem inteiros positivos m e n para os quais m2−n2 = 1.

Exemplo-65:

Seja p um número primo, p > 3. Sabe-se que para um certo inteiro positivo n o número pn possui20 dígitos, quando escrito na base 10.Prove que dentre esses dígitos existem no mínimo três iguais.

Solução

Depois de ler o problema (se preciso for, mais de uma vez, talvez até em voz alta), de forma a saberexatamente quais as quantidades dadas, quais as condições dadas (hipóteses) e o que se pede, vamos traçarum plano para a solução.

(KMO- 1981)Suponha o contrário. Isto é, dentre os 20 dígitos do número pn não existem três iguais. Istosignifica que um dos dígitos 0,1,2,3 · · · ,9 é usado na representação decimal de pn exatamente duasvezes. Assim, a soma dos dígitos de pn é igual a

2× (0+1+2+ · · ·+9) = 90.

Logo, pn é divisível por 3. Mas, isso é uma contradição, pois p é um primo maior do que 3, o queimplica que pn não pode ser divisível por 3. Portanto, dentre os dígitos de pn existem no mínimo trêsiguais.

Exemplo-66:

Prove que o polinômio P(x) = x5 − x2 + 2x− 1, com coeficientes inteiros, não admite raízesnegativas.

Solução

Depois de ler o problema (se preciso for, mais de uma vez, talvez até em voz alta), de forma a saberexatamente quais as quantidades dadas, quais as condições dadas (hipóteses) e o que se pede, vamos traçarum plano para a solução.

Suponha por absurdo que o polinômio P(x) dado admite uma raiz negativa. Seja r < 0 esta raiz. Assim,temos:

P(r) = r5− r2 +2r−1 = 0 r5 = r2−2r+1 = (r−1)2. (∗)

Agora, observe que o lado direito da igualdade (∗) é positivo, enquanto o lado esquerdo é negativo.Contradição, que decorre do fato de termos assumido que o polinômio dado admitia uma raiz negativa.Portanto, o polinômio P(x) = x5− x2 +2x−1, com coeficientes inteiros, não admite raízes negativas.

Page 59: NOTAS DE AULA - UFRGS

2.9 Redução ao Absurdo. 59

Exemplo-67:

Um tabuleiro 6×6 pode ser coberto por 18 dominós 2×1, de modo que cada dominó cubraperfeitamente duas casas do tabuleiro.Prove que, não importa como fazemos a cobertura (dentro das hipóteses), podemos traçar uma retaparalela a um dos bordos dividindo a cobertura em duas partes e tal que nenhum dominó fiquecortado ao meio por esta reta.

Solução

Depois de ler o problema (se preciso for, mais de uma vez, talvez até em voz alta), de forma a saberexatamente quais as quantidades dadas, quais as condições dadas (hipóteses) e o que se pede, vamos traçarum plano para a solução.

Observe que, como o tabuleiro é 6×6, existem 5 retas horizontais e 5 retas verticais dividindo otabuleiro, veja figura a seguir.

Não importa qual seja a cobertura feita, cada uma destas retas corta um número par de dominós. De fato,se uma dessas retas cortar um número ímpar de dominós, o tabuleiro ficará dividido em duas partes, cadauma delas conterá um número ímpar de metades de um dominó. Mas, isto é impossível, pois cada umadessas partes possui um número inteiro de casas cobertas por dominós e existem 36 casas no tabuleirotodo. Portanto, toda reta (paralela a um dos bordo do tabuleiro) corta um número par de dominós, de modoque os arranjos dos 18 dominós sobre as dez linhas sejam tais que no mínimo uma reta não corta ao meioqualquer dominó.

Page 60: NOTAS DE AULA - UFRGS

60 Capítulo 2. ESTRATÉGIAS

2.10 Suponha o problema resolvido (ou olhe de trás para diante)Uma tática que pode ser utilizada em jogos ou problemas em que se tem de construir alguma figura, é suporo problema resolvido. Então você poderá reverter seus passos e, portanto, construir uma solução para oproblema original.

Exemplo-68:

Dois amigos, A e B se divertem com o seguinte jogo, em que jogam alternadamente. O primeiro ajogar, A, escolhe um inteiro qualquer de 1 a 11, inclusive, e comunica ao segundo jogador, B, asua escolha e este soma-o à qualquer número de 1 a 11, inclusive, por ele escolhido, falando oresultado ao primeiro jogador que, na sua vez de jogar, o soma à qualquer número escolhido de 1a 11, e assim eles vão jogando, alternadamente, até que um deles obtenha o número 56, vencendoo jogo.Quem vence: A ou B? Qual é a estratégia para vencer?

Solução

Depois de ler o problema (se preciso for, mais de uma vez, talvez até em voz alta), de forma a saberexatamente quais as quantidades dadas, quais as condições dadas (hipóteses) e o que se pede, vamos traçarum plano para a solução.

Depois de ler o problema (se preciso for, mais de uma vez, talvez até em voz alta), de forma a saberexatamente quais as quantidades dadas, quais as condições dadas (hipóteses) e o que se pede, vamos traçarum plano para a solução.

O jogador A, que começa, vence.A estratégia do jogador A é olhar para a situação no final do jogo. Assim, na sua penúltima jogada, ojogador A tem de deixar para B uma soma menor do que 56−11 = 45, para garantir que B não possaatingir o número 56. Assim, ele deixa a soma igual a 44. Na sua jogada imediatamente anterior ele deixapara o jogador B uma soma menor do que 44−11 = 33, para garantir que o jogador B não possaatingir 44. Desse modo, as jogadas vencedoras de A são de tal maneira que os totais atingidos por elessejam: 8, 20, 32, 44 e 56.

Exemplo-69:

Dois amigos, A e B, disputam seguinte jogo, em que jogam alternadamente, usando um monte com27 de caroços de feijão. O jogador A começa. Uma jogada consiste em retirar 1, 2, 3 ou 4caroços do monte. O jogo termina quando todos caroços forem removidos. O vencedor é aquele quepossuir um número par de caroços quando todos os caroços tiverem sido retirados.Quem vence: A ou B? Qual a estratégia vencedora?

Solução

Depois de ler o problema (se preciso for, mais de uma vez, talvez até em voz alta), de forma a saberexatamente quais as quantidades dadas, quais as condições dadas (hipóteses) e o que se pede, vamos traçarum plano para a solução.

(Solução por David Angell, University of New South Wales, Australia. Fonte: ANGELL, D]Descreva a posição do jogo num momento qualquer por um número n par:• (n, I), se existem no monte n caroços de feijão e se o jogador que vai fazer seu movimento possui um

número ímpar de caroços ou• (n,P) se existem no monte n caroços de feijão e se o jogador que vai fazer seu movimento possui um

número par de caroços.Dizemos que uma posição é uma posição vencedora se o jogador que tem a vez de jogar pode garantir quevai vencer, não importando como o outro jogue. Assim, temos:• (1, I) é uma posição vencedora, porque o jogador que vai fazer seu movimento retira o último

caroço e fica com um número par deles.

Page 61: NOTAS DE AULA - UFRGS

2.10 Suponha o problema resolvido (ou olhe de trás para diante) 61

• (1,P) é uma posição perdedora, pois o jogador que vai fazer seu movimento retira o último caroço efica com um número ímpar deles.

• (2, I) é uma posição vencedora, porque retirando um caroço fica com um número par deles e deixa ooponente na posição (1,P), que é uma posição perdedora..

• (2,P) é uma posição vencedora, pois o jogador que vai fazer seu movimento retira os dois últimoscaroços e termina com um número par deles.

• Por razões análogas, (3, I),(3,P),(4, I) e (4,P) são posições vencedoras. Por exemplo, na posição(3,P) o oponente possui um número par de caroços, retirando dois caroços coloca o oponente naposição (1,P), que, como vimos acima, é uma posição perdedora.

• (5, I) é uma posição perdedora, pois o oponente possui um número ímpar de caroços, pois qualquermovimento deixa-o em uma das posições (4, I) ou (3, I) ou (2, I) ou (1, I).

• (5,P) é uma posição vencedora, porque retirando 4 caroços coloca o oponente na posiçãoperdedora (1,P).

• (6, I) é uma posição perdedora porque o oponente possui um número par de caroços, e qualquermovimento leva-o para uma das posições vencedoras (5,P) ou (4,P) ou (3,P) ou (2,P).

• (6,P) é uma posição vencedora, pois retirando 1 caroço coloca o oponente na posição perdedora(5, I). Se você continuar com esse raciocínio, vai descobrir que agora a situação se repete: asposições vencedoras e perdedoras para n caroços são as mesmas que para n−6 caroços. Porexemplo, se n = 7, então (n, I) vence e (n,P) perde, mesmo para n = 1; se n = 8, então ambasas posições (n, I) e (n,P) vencem; o mesmo para n = 2, e assim por diante.

Portanto, para n = 27, começando com um número par de caroços (0 é um número par) é essencialmenteo mesmo que a posição (3,P) e o primeiro jogador vence retirando dois caroços.A estratégia vencedora é simplesmente seguir a tabela de instruções dada abaixo. Como existem realmentesomente doze posições de (1, I) a (6,P), e três delas são posições perdedoras, segue que não importacomo você jogue, existem nove posições vencedoras para memorizar. Essas posições vencedoras estão natabela abaixo, onde n (mod 6) significa o resto quando n é dividido por 6.

n (mod 6) 0 1 2 3 4 5n Impar nenhum 1(2) 1 3(4) 3 nenhum

n Par 1 nenhum 2(3) 2 4 4

Page 62: NOTAS DE AULA - UFRGS

62 Capítulo 2. ESTRATÉGIAS

2.11 O Princípio da Indução.

Em muitos problemas da Matemática precisamos verificar a veracidade de uma afirmação, A(n), quedepende de um número natural n. Para fazer a verificação, usamos o método de indução.

Em que consiste o método de indução matemática?

A prova por indução pode ser pensada como a brincadeira de arrumar dominós em fila e derrubá-los comouma onda: derrubamos a primeira peça, que ao cair bate na segunda derrubando-a, que ao cair bate naterceira derrubando-a e assim por diante, até que todas elas sejam derrubadas, conforme mostra a Figura 1a seguir.

88

87 86 85848382818079787776757473727170696867666564636261605958575655545352515049484746454443424140393837363534333231302928272625242322212019181716151413121110987654

32

1

Figura 1 - Dominós em queda - Fonte:http://tex.stackexchange.com/a/151474

Agora, em vez de peças de dominós, pense numa seqüência de afirmações: A(1),A(2),A(3), . . . ,A(n), · · · .Imagine que seja possível provar as duas etapas seguintes:

1. a primeira afirmação, A(1), é verdadeira;2. sempre que uma afirmação for verdadeira, a imediatamente seguinte também é verdadeira.

Concluímos que todas as afirmações A(1),A(2),A(3), . . . ,A(n), · · · são verdadeiras.

Relacionando com as peças de dominó,1. seria a derrubada da primeira peça,2. seria a queda de uma peça de dominó provocada pela queda da peça anterior.

A parte 1. é chamada a hipótese de indução ou a base da indução e a 2. é a etapa indutiva.

zObservação. Veja que é natural aceitarmos o método de indução como um princípio, pois o queele garante a cerca de uma afirmação A(n) é que: A(1) verdadeira =⇒ A(2) verdadeira =⇒ A(3)verdadeira =⇒ A(4) verdadeira =⇒ . . . =⇒ A(n) verdadeira =⇒ A(n+1) verdadeira =⇒ . . .. Ouseja, essa sequência de implicações varre todos os números naturais.

Page 63: NOTAS DE AULA - UFRGS

2.11 O Princípio da Indução. 63

Exemplo-70:

(A soma dos primeiros n números naturais)Provar que, para todo número natural n, temos:

1+2+3+ . . .+n =n(n+1)

2

Solução

Observe que a afirmação A(n) a ser provada é: 1+2+3+ . . .+n = n(n+1)2 , para cada número natural

n.Assim, A(1) será entendida como a afirmação: 1 = 1×(1+1)

2 ;A(2) : 1+2 = 2×(2+1)

2 ;A(3) : 1+2+3 = 3×(3+1)

2 , e assim por diante.Etapa 1 – Vamos verificar a base da indução. Isto é, (i), que é o mesmo que verificar que A(1) éverdadeira. Para isso, basta observar que: 1 = 1×(1+1)

2 .

Etapa 2 – Vamos supor que para n = k, onde k um número natural maior do que ou igual a 1,a fórmula dada seja verdadeira. Isto é, 1+ 2+ 3+ . . .+ k = k(k+1)

2 . Agora, provaremos que paran = k + 1 a igualdade também se verifica. De fato, somando k + 1 em ambos os membros daexpressão anterior, obtém-se:

1+2+3+ . . .+ k+(k+1) =k(k+1)

2+(k+1) =

k(k+1)+2(k+1)2

=(k+1)× (k+2)

2.

Esse valor coincide com o valor da fórmula dada ao substituirmos n por k+1, pois:

1+2+3+ . . .+ k+(k+1) =(k+1).[(k+1)+1]

2=

(k+1)(k+2)2

.

Portanto, pelo Princípio da Indução, completamos a prova.

Exemplo-71:

(A soma dos quadrados dos primeiros n números naturais)Provar que, para todo número natural n, temos:

12 +22 +32 + . . .+n2 =n(n+1)(2n+1)

6

Solução

Observe que a afirmação A(n) a ser provada é: 1+2+3+ · · ·+n = n(n+1)2 , para cada número natural

n.Assim, A(1) será entendida como a afirmação: 1 = 1×(1+1)

2 ;A(2) : 1+2 = 2×(2+1)

2 ;A(3) : 1+2+3 = 3×(3+1)

2 , e assim por diante.Etapa 1 – Vamos verificar a base da indução. Isto é, (i), que é o mesmo que verificar que A(1) éverdadeira. Para isso, basta observar que: 1 = 1×(1+1)

2 .

Etapa 2 – Vamos supor que para n = k, onde k um número natural maior do que ou igual a 1,a fórmula dada seja verdadeira. Isto é, 1+ 2+ 3+ · · ·+ k = k(k+1)

2 . Agora, provaremos que paran = k + 1 a igualdade também se verifica. De fato, somando k + 1 em ambos os membros daexpressão anterior, obtém-se:

1+2+3+ . . .+ k+(k+1) =k(k+1)

2+(k+1) =

k(k+1)+2(k+1)2

=(k+1)× (k+2)

2.

Page 64: NOTAS DE AULA - UFRGS

64 Capítulo 2. ESTRATÉGIAS

Esse valor coincide com o valor da fórmula dada ao substituirmos n por k+1, pois:

1+2+3+ . . .+ k+(k+1) =(k+1).[(k+1)+1]

2=

(k+1)(k+2)2

.

Portanto, pelo Princípio da Indução, completamos a prova.

Exemplo-72:

(A soma dos quadrados dos primeiros n números naturais)Provar que, para todo número natural n, temos:

12 +22 +32 + . . .+n2 =n(n+1)(2n+1)

6

Solução

A afirmação, A(n), a ser provada é: 12 +22 +32 + . . .+n2 = n(n+1)(2n+1)6 , para todo número natural

n. Assim,A(1) será entendida como a afirmação: 1×(1+1)×(2×1+1)

6 = 2×36 = 1 = 12;

A(2) : 12 +22 = 2×(2+1)×(2×2+1)6 ;

A(3) : 12 +22 +32 = 3×(3+1)×(2×3+1)6 , e assim por diante.

Etapa 1 – Vamos verificar a base da indução, isto é, (i). Para isso, basta observar que: 12 =1×(1+1)×(2×1+1)

6 = 2×36 = 1.

Etapa 2 – Vamos supor que, para n = k, onde k é um número natural maior do que ou igual a 1, afórmula dada seja verdadeira. Isto é, 12 +22 +32 + . . .+ k2 = k(k+1)(2k+1)

6 para todo número naturalk.Agora, provaremos que para n = k+1 a igualdade também se verifica. De fato, somando (k+1)2

em ambos os membros da igualdade anterior, obtém-se:

12 +22 +32 + . . .+ k2 +(k+1)2 =k(k+1)(2k+1)

6+(k+1)2 = (k+1)× k(2k+1)

6+(k+1)2 =

= (k+1)× 2k2 + k+6k+66

= (k+1)×2(k+ 3

2 )(k+2)6

=

=(k+1)(2k+3)(k+2)

6=

(k+1)(k+2)[2(k+1)+1]6

,

que é a expressão n(n+1)(2n+1)6 quando n = k+ 1, o que completa a prova. Observe que na anti-

penúltima igualdade acima foi usado que o trinômio do segundo grau 2k2 + 7k+ 6 se fatora em2(k+2)(k+ 3

2 ).

Exemplo-73:

(A soma dos cubos dos primeiros n números naturais)Provar que, para todo número natural n, temos:

13 +23 +33 + . . .+n3 =

[n(n+1)

2

]2

Solução

Page 65: NOTAS DE AULA - UFRGS

2.11 O Princípio da Indução. 65

A afirmação, A(n), a ser provada é: 13 +23 +33 + . . .+n3 = [ n(n+1)2 ]2, para todo número natural

n. Assim, A(1) será entendida como a afirmação:[

1×(1+1)2

]2= 2

2 = 1 = 13; A(2) : 13 +23 = 9 =[2×(2+1)

2

]2; A(3) : 13 +23 +33 = 36 =

[3×(3+1)

2

]2, e assim por diante.

Etapa 1 – Vamos verificar a base da indução. Isto é, vamos verificar que (i) é verdadeira. Para isso,basta observar que: [1(1+1)

2

]2=

[22

]2

= 12 = 1 = 13.

Etapa 2 – Vamos supor que para n = k, onde k é um número natural maior do que ou igual a 1, afórmula dada seja verdadeira. Isto é, 13 +23 +33 + · · ·+ k3 = [ k(k+1)

2 ]2, para todo número natural k.Agora, provaremos que para n = k+1 a igualdade também se verifica. De fato, adicionando (k+1)3

a ambos os membros, temos:

13 +23 +33 + · · ·+ k3 +(k+1)3 =

[k(k+1)

2

]2

+(k+1)3 =

k2(k+1)2

4+(k+1)3 =

k2(k+1)2 +4(k+1)3

4=

(k+1)2[k2 +4(k+1)

]4

=

=(k+1)2[k+2]2

4=

[(k+1)(k+2)

2

]2

,

que é a fórmula para n = k+1, o que completa a prova.

zNota - Observe que, pelo exemplo 1.3.1, temos que 1+ 2+ 3+ · · ·+ n = n(n+1)2 . Agora, o

exemplo 1.3.3, curiosamente, nos permite concluir que:

13 +23 +33 + . . .+n3 = (1+2+3+ . . .+n)2 =

[n(n+1)

2

]2

,

um resultado, convenhamos, surpreendente!

Exemplo-74:

Têm-se 2n moedas de ouro, sendo uma delas falsa, com peso menor que as demais. Dispõe-sede uma balança de dois pratos, sem nenhum peso. Mostre, por indução, que é possível achar amoeda falsa com n pesagens.

Solução

Para n = 1 teremos 2n = 21 = 2 moedas. Neste caso colocamos uma moeda em cada prato dabalança. Aquele prato que ficar mais alto conterá a moeda mais leve e, portanto, com apenas umapesagem conseguimos, nesse caso, descobrir a moeda falsa.Suponhamos que a proposição seja verdadeira para o número natural n = k, isto é, com 2k moedas,uma quantidade de k pesagens são suficientes para descobrirmos a moeda falsa (Hipótese da indução).Agora, vamos mostrar que a proposição é válida para n = k+1, ou seja, com 2k+1 moedas podemosdescobrir a moeda falsa com k+1 pesagens. Com efeito, se possuirmos 2k+1 moedas podemosdividir todas essas moedas em dois grupos com 2k moedas cada um, pois 2k+1 = 2.2k. Colocamosentão cada um desses dois grupos nos pratos da balança. Dessa forma, um dos pratos da balança iráficar mais alto (pois a moeda falsa estára num desses dois grupos e portanto há um grupo mais leve queoutro). O prato da balança que ficar mais alto acusará em qual dos grupos estará a moeda falsa. Noteque agora temos um grupo com 2k moedas, onde necessariamente está a moeda falsa. Pela hipóteseda indução, com k pesagem podemos descobrir qual é a moeda falsa. Assim, com um grupo de 2k+1

moedas, onde apenas uma é falsa (mais leve), k+1 pesagem são suficientes para descobrir qual é a

Page 66: NOTAS DE AULA - UFRGS

66 Capítulo 2. ESTRATÉGIAS

moeda falsa; uma pesagem para descobrir qual o grupo em que está a moeda falsa e k pesagens paradescobir qual é, de fato, a moeda falta, resultanto então em um total de k+1 pesagens.

Exemplo-75:

Um grupo de n≥ 2 pessoas está em uma fila para comprar entradas para o cinema. A primeirapessoa da fila é uma mulher e a última é um homem. Mostre que, em algum ponto da fila, umamulher está diretamente na frente de um homem.

Solução

Para n = 2, teremos uma fila com apenas duas pessoas, na qual a primeira pessoa da fila é uma mulhere a segunda é um homem, o que revela que há na fila uma mulher diretamente à frente de um homem.Suponhamos que a proposição seja verdadeira para o número natural n = k, ou seja, numa fila com kpessoas em que a primeira pessoa é uma mulher e a última é um homem em algum ponto da fila,umamulher está diretamente na frente de um homem (hipótese da indução).Agora vamos demonstrar que a proposição é verdadeira para n = k+1, ou seja, que numa fila comk+1 pessoas em que a primeira pessoa é uma mulher e a última é um homem em algum ponto dafila,uma mulher está diretamente na frente de um homem. De fato, seja

a1a2a3 . . .akak+1

uma fila com k+1 pessoas em que a primeira pessoa da fila é uma mulher e a última um homem.Se a pessoa a2 for um homem a proposição foca demonstrada, pois a1 é uma mulher e aí teremosentão uma mulher diretamente à frente de de um homem. Se a2 for uma mulher, teremos então umafila com k pessoas, a2a3 · · ·akak+1 onde a primeira pessoa da fila é uma uma mulher e a última éum homem, que pela hipótese da indução, satisfaz a proposição. Assim, de qualquer forma temos oresultado desejado, ou seja, que numa fila com k+1 pessoas, em que a primeira pessoa é uma mulhere a última é um homem em algum ponto da fila,uma mulher está diretamente na frente de um homem.

Exemplo-76:

Se x+ 1x é um número inteiro, mostre que xn + 1

xn é número inteiro para todo n ∈ N.

Solução

Para n = 1 a proposição é claramente verdadeira, pois para n = 1 a expressão xn + 1xn transforma-se

em x+ 1x , que por hipótese, é um número inteiro.

Usando o princípio da indução segunda forma, suponhamos que a expressão xn + 1xn , seja um número

inteiro para n = 2,3, · · · ,k (hipótese da indução).Agora vamos provar que para n = k+1 a expressão xn + 1

xn é um número inteiro. Com efeito, temosque (

x+1x

)(xk +

1xk

)= xk+1 +

1xk+1 + xk−1 +

1xk−1 ⇒

xk+1 +1

xk+1 =

(x+

1x

)(xk +

1xk

)−(

xk−1 +1

xk−1

)ora, como x+ 1

x é, por hipótese inteiro, e xn + 1xn , é um número inteiro para n = 2,3, · · · ,k, segue que

no segunto membro da igualdade

xk+1 +1

xk+1 =

(x+

1x

)(xk +

1xk

)−(

xk−1 +1

xk−1

)todos os números são inteiros, o que revela que a expressão xk+1 + 1

xk+1 é um número inteiro, o quedemonstra o resultado desejado.

Page 67: NOTAS DE AULA - UFRGS

3. Problemas Diversos

Exemplo-77:

Começando com um hexágono regular, uma forma geométrica é gerada em estágios. Na Fase1, há um hexágono. Cercando-o completamente, adicionando um hexágono regular congruenteao incial, para cada lado dele exposto, temos que na Fase 2, existem 7 hexágonos no total.Cercando completamente a nova forma pela adição de um hexágono regular, congruentes aosoutros, para cada lado exposto, temos que na Fase 3 existem um total de 19 hexágonos, vejaFigura a seguir.

Encontre uma fórmula para o número total de hexágono em Fase n.

Solução

(AMO) No hexagono inicial, traçamos as três maiores diagonais e as prolongamos. Elas cortam afigura em 6 partes congruentes, veja Figura a seguir.

Page 68: NOTAS DE AULA - UFRGS

68 Capítulo 3. Problemas Diversos

Agora, omitindo o hexágono central, observa-se que a quantidade de hexágono em cada partecongruente e em cada estágio é:

1,(

12+1+

12

), 3,

(12+3+

12

), 5,

(12+5+

12

), 7, . . .

Isto é, a quantidade de hexágonos em cada parte congruente e em cada estágio é:

1, 2, 3, 4, 5, 6, 7, . . . , n−1 (depois de n estagios)

Portanto, se Tn é o número de hexágonos no estágio n, temos que

Tn = 6[1+2+3+4+5+ · · ·+(n−1)]+1 =6n(n−1)

2+1 = 3n(n−1)+1 = 3n2−3n+1.

Exemplo-78:

Sejam m e n números inteiros positivos e um tabuleiro m×n, onde cada casa corresponde aum quadrado 1×1. No tabuleiro, existem quantos retângulos distintos, cada um deles tendoos lados paralelos aos bordos?

Solução

Observe que cada retângulo é definido por duas retas horizontais e duas retas verticais. Por outro lado,no tabuleiro dado existem m+1 retas horizontais e n+1 retas verticais, veja a Figura a seguir parao caso em que m = 7 e n = 5.

Para formar um retângulo nas condições do problema, temos que escolher duas retas horizontais e duasretas verticais. Agora, observe que as retas retas horizontais são escolhidas de forma independentedas retas verticais e a ordem pelas quais se faz a escolha das duas retas horizontais e das duas retasverticais são irrelevantes. Ainda, não são permitidas a escolha de retas repetidas, pois, caso contrário,

Page 69: NOTAS DE AULA - UFRGS

69

teríamos algum retângulo com área zero.A escolha de duas retas horizontais distintas dentre as dentre m+1 existentes se dá de

(m+12

)e a

escolha de duas retas verticais distintas dentre as dentre n+1 existentes se dá de(n+1

2

). Pelo Princípio

Multiplicativo da Análise Combinatória, a resposta é(m+1

2

)×(n+1

2

).

Exemplo-79:

Num tabuleiro de xadrez (8×8), quantos cavalos (peça do jogo de xadrez) podemos arrumarde modo que dois quaisquer deles não se ameacem mutuamente?

Solução

Inicialmente, observe que, no jogo de xadrez, o cavalo, que é a peça mais "ágil", se movimenta emforma de L: de uma casa sobre a coluna em que se encontra para a casa localizada contando-se duascasas (para cima ou para baixo) na mesma coluna e uma casa (para a esquerda ou para direita) na linhaperpendicular (ou contando-se duas casas (para esquerda ou para direita) na mesma linha e uma casa(para cima ou para baixo) na coluna perpendicular. Isto é, o cavalo se movimenta de um canto parao canto oposto de um retângulo 2×3 ou 3×2. Por exemplo, se o cavalo está na casa d4 (istoé, a casa que se encontra na interseção da coluna d com a linha 4) em um movimento, existem 8possíveis casas para ele ocupar. Observe que o cavalo se movimenta no espaço!, veja Figura AP8 aseguir.

Figura AP8- Movimentos do cavalo

Observe também que um movimento de um cavalo se dá sempre de uma casa para outra de cor oposta,veja Figura a seguir.

Figura - Em um movimento, o cavalo se desloca para uma casa de cor oposta

Page 70: NOTAS DE AULA - UFRGS

70 Capítulo 3. Problemas Diversos

Isto significa que, em um único movimento, o cavalo sai de uma casa de cor branca para uma casa decor preta, e vice versa.Logo, se o cavalo está numa casa branca ele só pode ameaçar peças que estejam em casas de corpreta. Deste modo, como o tabuleiro possui 32 casas brancas, podemos colocar 32 peças sem que seameacem mutuamente. Por outro lado, se colocamos 33 cavalos, um deles estará obrigatoriamentenuma casa preta e, neste caso, existirá um cavalo, localizado numa casa branca que o ameaçará.Portanto, a resposta é 32.

Exemplo-80:

Juliette e Philippe disputam um jogo em que fazem seus movimentos alternadamente. No iníciodo jogo, cada vértice de um quadrado é coberto por um número de fichas. Um movimentoconsiste em cada jogador escolher um lado do quadrado e retirar de suas extremidades tantasfichas quanto ele desejar, desde que ele retire pelo menos uma.

BA

D C

••

• •

Não é necessário retirar o mesmo número de fichas de cada extremidade. O jogador que retirara última ficha vence o jogo. No início do jogo, existem 10 fichas no vértice A, 11 fichas novértice B, 12 fichas no vértice C e 13 fichas no vértice D.Se Juliette começa, qual é a sua estratégia para vencer?

Solução(CM, 37: No 5 September 2011) Sejam a,b,c e d o número de fichas nos vértices A,B,C e D,

respectivamente. A estratégia de Juliette é fazer seus movimentos para ter a = c e b = d.Se na sua vez de jogar, Philippe recebe um quadrado com a = c e b = d, então ele retira no mínimouma ficha, e não pode retirar fichas de ambos os extremos de uma diagonal. Portanto, ele sempre passapara Juliette um quadrado com a 6= c e b 6= d. Se Juliette recebe um quadrado a 6= c e b 6= d, entãopara cada diagonal ela deve escolher a extremidade com maior número de fichas. Nesse caso, ela deveescolher o lado que liga essas duas extremidades e retirar fichas até que a = c e b = d. Assim, elasempre pode aplicar essa estratégia e obter a = c = 0 e b = d, vencendo o jogo.

Exemplo-81:

Tem-se cartões sobre uma mesa de modo que em cada um deles está escrito um par de númerosinteiros positivos. Existe exatamente um cartão para cada par de números inteiros, (a,b),com 1≤ a < b≤ 2003. Dois jogadores disputam um jogo, em que jogam alternadamente.Cada movimento corresponde a retirar um cartão e escrever no quadro-negro o produto abdos números lá escritos. O primeiro jogador que faz com que o Máximo Divisor Comumdos números no quadro-negro seja igual a 1 perde.Qual dos jogadores tem uma estratégia vencedora: o primeiro ou o segundo a jogar?

Solução(OMM - 2003) O primeiro jogador tem uma estratégia vencedora.

Considere os números no quadro-negro imediatamente antes do movimento perdedor. O MáximoDivisor Comum de todos esses números deve ser um inteiro d > 1.

Page 71: NOTAS DE AULA - UFRGS

71

Se d não é um número primo, então ele possui algum divisor, k, menor do que ele, de modo que ocartão com o par (1,k) ainda pode ser retirado. Contradição, pois o próximo movimento é perdedor.Logo, d é um número primo e todo cartão com um par (a,b), com d dividindo ab deve ter jásido retirado. Existem 2002 pares da forma (a,d), pois, pela hipótese do problema, a pode serqualquer um dos números 1, 2, 3, . . . 2003 exceto d. De maneira análoga, existem 2002 pares daforma (a,2d), com 2d ≤ 2003. Agora, observe que, deste modo, contamos duas vez o par (d,2d).Logo, d tem de ser escolhido de modo que 2d < 2003 < 3d, o que implica que existem 4003 parespossíveis.O primeiro jogador pode vencer se ele retira o par (1,997). ssim, as possíveis jogadas são:(k,997), para k = 2, 3, 4, . . . , 996, 998, . . . , 2003 (que são 2001 possibilidades), e (k,1994),para k = 1, 2, 3, . . . , 996, 998, 999, , 1993, 1995, 1996, . . . , 2003 (que são 2001 possibilidades).Portanto, existe um número par de movimentos disponíveis e o primeiro jogador vencerá. Observeque o segundo jogador só pode reduzir o número de movimentos disponíveis se ele perder.

Exemplo-82:

Num hotel os quartos estão dispostos como um tabuleiro 2× 8, estando cada um delesocupado por um hóspede. Todos os hóspedes estão se sentindo desconfortáveis em seusaposentos, de modo que cada um deles gostaria de passar para um quarto adjacente, horizontalou verticalmente. Naturalmente, eles podem fazer a mudança simultaneamente, de tal maneiraque cada quarto fique novamente ocupado por um hóspede.De quantas maneiras distintas podem ser feitos esses movimentos coletivos?

Solução

(HMITMT - 2003) Imagine que os quartos são pintados alternadamente de preto e branco, no estilo deum tabuleiro de xadrez, veja figura a seguir.

Pela figura, é fácil ver que, cada hóspede se movimenta de um quarto preto para um quarto adjacentebranco, e vice-versa. Se para um hóspede, que inicialmente está num quarto preto, colocamos umdominó sobre o quarto antigo e o novo, obtemos uma pintura do tabuleiro do tipo 2×n, formada porn dominós, pois cada quarto preto é usado uma vez e cada quarto branco é usado uma vez. Aplicandoprocedimento análogo para cada hóspede que inicialmente está num quarto branco e se movimentapara um quarto preto, obtemos uma segunda pintura para o tabuleiro. Inversamente, é fácil verificarque, para qualquer par de quartos dessas pinturas, determina um padrão de movimento. Também éfácil provar por indução que o número de pinturas do tipo dominós de um tabuleiro de dimensões2×n é (n+1)-ésimo número de Fibonacci.Isto é fácil de ver para os casos de n = 1,2 e para um retângulo 2×n, onde os dois quadrados mais adireita pertencem a um dominó vertical ou deixam um retângulo de dimensão 2× (n−1) ser pintadoarbitráriamente, ou dois dominós ocupam quartos adjacentes, deixando um retângulo de dimensão

Page 72: NOTAS DE AULA - UFRGS

72 Capítulo 3. Problemas Diversos

2× (n−2) ser pintado livremente. Portanto, o número de pinturas possíveis é o número de Fibonacci.Deste modo, o número de pinturas de um tabuleiro de dimensão 2×8 é 34, e o número possível depares de tais pinturas é igual a 342 = 1156, que é a resposta do problema.

Exemplo-83:

Sem levar em consideração a ordem das parcelas, de quantas maneiras podemos expressar2002 como soma de 3 inteiros positivos?(Consideramos 2 + 1000 + 1000 e 1000 + 2 + 1000 como sendo a mesma forma deexpressar 2002 como soma de 3 inteiros positivos)

Solução

(HMITMT - 2002) Sejam a,b e c três números positivos para os quais a+ b+ c = 2002. Parafacilitar o entendimento e evitar redundancia, vamos supor que a≤ b≤ c. Observe que, para cadaescolha de a e b, existe uma única escolha para c, pois c = 2002− (a+b). Agora, vamos contartodas as possibilidades para a e b considerando dois casos:Caso 1 O número a é par.Caso 2 O número a é ímpar.Suponha que o número a do terno procurado seja um número par. Neste caso, para cada valor de a,teríamos b = c = 2002−a

2 , que é um número inteiro. Com isso, teríamos a+ 2002−a2 + 2002−a

2 = 2002,satisfazendo ao problema. Observe que, para b = 2002−a

2 ≥ a, segue que 2002−a ≥ 2a, que é omesmo que a≤ 667. Como a é um número par, temos 2≤ a≤ 666.Para cada valor de a, existem ( 2002−a

2 − a+ 1) valores para b. Ou seja, para cada valor de a,existem 2002−a−2a+2

2 = 1002− 3a2 valores para b.

Agora, se a = 2, temos 1002− 3×22 = 999 valores para b. Se a = 4, temos 2002− 3×4

2 = 996valores para b. Se a = 6, temos 2002− 3×6

2 = 993 valores para b, e assim por diante até a = 666,que nos dá 2002− 3×666

2 = 3 valores para b. Para calcular a quantidade de todos os números a,b,cpossíveis, quando a é um número par, somamos os números obtidos anteriormente, que formam umaprogressão aritmética. Isto é, a quantidade de valores possíveis para os ternos de número a,b,c, coma par, satisfazendo ao problema é igual a: 999+996+993+ ...+3 = 999+3

2 ×333 = 501×333 =166833.Seja a um número ímpar. Neste caso, para cada valor de a, existem [ (2002−(a+1)

2 − a+ 1] =[ (2002−a−1−2a+2

2 ] = [ 2000−3(a−1)2 ] = 1000− [ 3(a−1)

2 ] possíveis valores para o número b.Como dado o número a, tem-se: b = 2002−(a+1)

2 ≥ a, segue que 2002− (a+1)≥ a, que é o mesmoque a≤ 667. Como a é um número ímpar positivo, temos 1≤ a≤ 667.Agora, se a = 1, temos que existem 1000− 3×(1−1)

2 = 1000 valores possíveis para b. Se a = 3,existem 1000− 3×(3−1)

2 = 997 valores possíveis para b. Se a = 5, existem 1000− 3×(5−1)2 = 994

valores possíveis para b, e assim por diante até a = 667, que nos dá 1000− 3×(667−1)2 = 1 valor

possível para b. Para calcular a quantidade de todos os números a,b,c possíveis, quando a é umnúmero ímpar, somamos os números obtidos anteriormente, que formam uma progressão aritmética.Isto é, a quantidade de valores possíveis para os ternos de número a,b,c, com a ímpar, satisfazendoao problema é igual a:1000+997+994+ . . .+1 = 1000+1

2 ×334 = 1001×167 = 167167.Portanto, existem 166833+ 167167 = 334000 possíveis arranjos dos números inteiros positivosa,b,c satisfazendo ao problema.

Page 73: NOTAS DE AULA - UFRGS

73

Exemplo-84:

Um tesouro está enterrado num tabuleiro de xadrez 8×8. Em cada uma das casas do tabuleiroexiste enterrado uma mensagem que indica o número mínimo de passos que se necessita parachegar a casa do tesouro. Necessita-se de um passo para se mover de uma casa para a casavizinha, que é uma que tem um lado comum com a casa de partida.Determine o número mínimo de casas que se deve escavar para chegar com certeza ao tesouro.

Solução

(TT - 2012) Se escolhemos uma casa qualquer para escavar, não há nenhuma garantia de que otesouro esteja lá. Se a mensagem retirada diz que o tesouro está num outro quadrado, fazendo uma sóescavação, não é possível determinar a exata localização dele. Vamos provar que, para encontrar otesouro, temos que escavar duas casas.Numeramos as linhas do tabuleiro 8×8, de baixo para cima, de 1 a 8 e as colunas, da esquerdapara a direita, de 1 a 8. Assim, cada casa do tabuleiro é perfeitamente identificada por um par denúmeros (b,c), com 1≤ b,c≤ 8, veja figura a seguir.

(1,1)

(2,1)

(3,1)

(4,1)

(5,1)

(6,1)

(7,1)

(8,1)

(1,2)

(2,2)

(3,2)

(4,2)

(5,2)

(6,2)

(7,2)

(8,2)

(1,3)

(2,3)

(3,3)

(4,3)

(5,3)

(6,3)

(7,3)

(8,3)

(1,4)

(2,4)

(3,4)

(4,4)

(5,4)

(6,4)

(7,4)

(8,4)

(1,5)

(2,5)

(3,5)

(4,5)

(5,5)

(6,5)

(7,5)

(8,5)

(1,6)

(2,6)

(3,6)

(4,6)

(5,6)

(6,6)

(7,6)

(8,6)

(1,7)

(2,7)

(3,7)

(4,7)

(5,7)

(6,7)

(7,7)

(8,7)

(1,8)

(2,8)

(3,8)

(4,8)

(5,8)

(6,8)

(7,8)

(8,9)

Linha 1

Linha 2

Linha 3

Linha 4

Linha 5

Linha 6

Linha 6

Linha 8

Col

una

1

Col

una

2

Col

una

3

Col

una

4

Col

una

5

Col

una

6

Col

una

7

Col

una

8

Escavamos as casas (1,1) e (8,1). Sejam a e b, respectivamente, os números que encontramos apósescavarmos as duas casas. Cada um desses números representa o comprimento do caminho mínimoque devemos fazer da casa escavada até o tesouro. Podemos imaginar, sem perda de generalidade, queo caminho mínimo realiza primeiro todos os movimentos verticais necessários em seguida e, após isso,os movimentos horizontais até atingir o tesouro.Vamos supor que o tesouro esteja na casa (i, j) do tabuleiro. Os caminhos mínimos para se atingir otesouro, a partir das casas escavadas, são de dois tipos:Tipo (I) - aquele que começa na casa (1,1), vai até a casa (1, j) e daí até a casa (i, j).Tipo (II) - aquele que começa na casa (8,1), vai até a casa (8, j) e daí até a casa (i, j).Deste modo, sejam V1 e H1 a quantidade de movimentos verticais e horizontais, respectivamente,a partir da casa (1,1) até chegar ao tesouro e V2 e H2 a quantidade de movimentos verticais ehorizontais a partir da casa (8,1) até chegar ao tesouro. Desse modo, temos que:{

V1 +H1 = aV2 +H2 = b

Pelo que indicamos acima, os movimentos horizontais dos dois caminhos são os mesmos, i. e.H1 = H2, e a diferença | a−b | é igual a diferença entre os comprimentos dos movimentos verticaisdos dois caminhos. Além disso, a soma dos comprimentos dos dois movimentos verticais é igual aV1 +V2 = 7.Portanto, supondo a≥ b, teremos um sistema de equações lineares 2×2:

Page 74: NOTAS DE AULA - UFRGS

74 Capítulo 3. Problemas Diversos{V1−V2 = a−b

V1 +V2 = bEste sistema tem solução única, o que determina exatamente a posição do tesouro.Para ilustrar, considere o tabuleiro a seguir, onde em cada está escrito o número obtido com a escavaçãoe a identificação da casa onde se encontra o tesouro.

8

7

6

5

6

7

8

9

7

6

5

4

5

6

7

8

6

5

4

3

4

5

6

7

5

4

3

2

3

4

5

6

4

3

2

1

2

3

4

5

3

2

1

X

1

2

3

4

4

3

2

1

2

3

4

5

5

4

3

2

3

4

5

6

Quando escavamos a casa (1,1) obtemos o número a = 8. Isto é, o caminho mínimo de (1,1) atéo tesouro tem comprimento 8. Quando escavamos a casa (8,1) obtemos o número b = 9. Ou seja,o caminho mínimo de (8,1) até o tesouro tem comprimento 9.Para se mover da casa (1,1) até o tesouro, localizado neste exemplo na casa (4,6), temos que fazerum movimento vertical até a casa (5,1) e de lá fazemos um movimento horizontal até a casa (5,6).Além disso, a diferença 9−8 = 1. Logo, temos que encontrar dois números (entre 1 e 8) cuja somaseja 7 e cuja diferença seja 1. Os únicos números que satisfazem são 3 e 4. Portanto, de (1,1)temos que subir 3 casas e, de (8,1), mover para baixo 4 casas. Como o comprimento do caminhomínimo de (1,1) até o tesouro é 8, temos que andar 5 casas horizontalmente. Portanto, o tesouroestá na casa (4,6).

Exemplo-85:

Num tabuleiro 8×8 comum, com as casas pintadas alternadamente de preto e branco, ummovimento permitido é trocar de posição quaisquer duas linhas ou colunas.Aplicando uma sequência de tais movimentos, é possível obter um tabuleiro onde a metade à es-querda seja totalmente formada por casas pretas e a metade à direita seja formada inteiramentepor casas brancas?

Solução

(LMO - 1987) Observe que, em qualquer movimento permitido, não se altera o número de quadradospretos numa coluna do tabuleiro, que inicialmente é quatro. Logo, nunca obteremos uma colunaconsistindo de oito quadrados pretos. Portanto, é impossível obter um tabuleiro onde a metade àesquerda seja totalmente formada por casas pretas e a metade à direita seja formada inteiramente porcasas brancas.

Page 75: NOTAS DE AULA - UFRGS

75

Exemplo-86:

Um mágico tem cem cartões numerados de 1 a 100. Ele coloca os cartões em três caixas,uma vermelha, uma branca, uma azul, de tal modo que cada caixa contém no mínimo umcartão. Uma pessoa da plateia escolhe duas dessas caixas e retira cartões de cada uma delas.Em seguida, revela a soma dos números daqueles cartões. Dada esta informação, o mágicolocaliza a caixa da qual nenhum cartão foi retirado.De quantas maneiras o mágico pode repartir os cartões nas caixas de modo que ele possasempre executar seu truque com êxito?(Duas maneiras de repartir os cartões são consideradas distintas se ao menos um cartão écolocado em duas caixas diferentes)

Solução

(OIM - 2000) A resposta é 12.Vamos chamar, respectivamente, de r,w,b as cores das caixas vermelha, branca, azul.Vamos dizer que a cor do número i seja a da cor da caixa em que ele se encontra e todo númeroconsiderado esteja no conjunto {1,2,3, . . . ,100}.Para que o truque funcione, sempre que x+ y = z+ t e os cartões de números x e y estejam emcasas distintas, então ou z e t estão nestas mesmas caixas ou ambos estão na caixa restante. Issosignifica dizaer que, nenhum par de número de cores distintas tem a mesma soma que outros dois emcaixas distintas.Consideramos dois casos:Caso 1: Suponhamos que existe uma caixa tal que os cartões numerados com i, i+1, i+2 são de coresdistintas, digamos vermelho (r), branco (w), azul (b), respectivamente.Questão: Em que caixa está o cartão de número i+3?Está na caixa vermelha. De fato, como i︸︷︷︸

r

+(i+3)︸ ︷︷ ︸?

= (i+1)︸ ︷︷ ︸w

+(i+2)︸ ︷︷ ︸b

, para que o truque funcione,

(i+3) tem de ser vermelho (r).De modo análogo, como (i+4)︸ ︷︷ ︸

?

+(i+1)︸ ︷︷ ︸w

= (i+3)︸ ︷︷ ︸r

+(i+2)︸ ︷︷ ︸b

, para que o truque funcione, segue que

(i+ 4) é branco (w). Do mesmo modo, como (i−1)︸ ︷︷ ︸?

+(i+2)︸ ︷︷ ︸b

= (i)︸︷︷︸r

+(i+1)︸ ︷︷ ︸w

, segue que (i− 1) é

branco (w)Com isso, concluímos que três cores vizinha distintas determinam a cor do próximo número. Alémdisso, este processo se repete: rwb é seguido por r,w,b e assim por diante. Este processo tambémfunciona inversamente: rwb é precedido por b, e assim por diante.Logo, é suficiente definir as cores dos cartões numerados com 1,2,3, que pode ser feito de 6 modosdistintos. Todos esses arranjos são tais que as somas dos números vermelhos e azuis, dos númerosbrancos e azuis, e dos números vermelhos e azuis, são restos distintos módulo 3. Isto é, a primeira caixacomtém os cartõe de números 1,4,7, · · · ,100, a segunda caixa contém os de números 2,5,8, · · · ,98 ea terceira contém 3,6,9, · · · ,99Caso 2: Suponhamos que não existem três cartões com números consecutivos em caixas distintas.Sem perda de generalidade, suponhamos que o número 1 seja vermelho (r).Agora, seja i o maior número que não é vermelho. Vamos supor que i seja branco. Seja k o menornúmero azul. Como, por hipótese, não existe três números consecutivos de cores distintas, rwb, temosnecessariamente que (i+1)< k.Suponhamos que k < 100. Como i︸︷︷︸

w

+ k︸︷︷︸b

= (i−1)︸ ︷︷ ︸r

+(k+1)︸ ︷︷ ︸?

, segue que (k+1) é vermelho (r).

No entanto, como i︸︷︷︸w

+(k+1)︸ ︷︷ ︸r

= (i+1)︸ ︷︷ ︸?

+ k︸︷︷︸b

, segue que (i+1) deve ser azul b, o que contradiz o

fato de que k é o menor número azul. Logo, k = 100.Do mesmo modo, como (i−1)︸ ︷︷ ︸

r

+ 100︸︷︷︸b

= i︸︷︷︸w

+ 99︸︷︷︸?

, segue que 99 é branco.

Page 76: NOTAS DE AULA - UFRGS

76 Capítulo 3. Problemas Diversos

Se t é um número inteiro, com t > 1 e o cartão numerado com t esteja na caixa vermelha (r), comot︸︷︷︸r

+ 99︸︷︷︸w

= (t−1)︸ ︷︷ ︸?

+ 100︸︷︷︸b

, temos que (t− 1) é azul (b), segue que o menor número azul é 100.

Portanto, a pintura dos números é feita do seguinte modo: rww · · ·wwb.Deste modo, se a soma é maior do que ou igual a 100, então a caixa restante é azul; se a soma é101, a caixa restante é branca; e se a soma é maior do que 101, a caixa restante é vermelha. Ouseja, uma caixa contém o cartão com número 1, uma outra caixa contém os cartões com os números2,3,4, · · · ,99 e a outra caixa contém somente o cartão com número 100. O número de tais arranjos é6.Portanto, a resposta do problema é 6+6 = 12.

Exemplo-87:

Provar ou dê um contra-exemplo: Dado um conjunto finito de pontos no plano comcoordenadas inteiras, é possível pintar de vermelho alguns destes pontos e os restantes debranco de tal forma que para qualquer reta L paralela a um dos eixos coordenados, onúmero de pontos vermelhos e o número de pontos brancos sobre L diferem por no máximo 1.

Solução

(OIM - 1986) A resposta é sim.Sejam P1,P2,P3, · · · ,Pn pontos sobre uma reta r paralela a um eixo, contados da esquerda para adireita e de cima para baixo. Desenhamos segmentos juntando P1 a P2, P3 a P4, · · · . De um modogeral, traçamos um segmento juntando os pontos P2i−1 a P2i. Feito isso para toda dessas retasr, obtemos uma algumas linhas poligonais. Se uma dessas poligonais é fechada, significa que elaforma um poligono com um número par de vértices. Agora, pintamos os vértices dessa poligonalalternadamente de vermelho e branco. Um ponto que não esteja sobre a poligonal pode ser pintado deforma arbitrária. A pintura obtida satisfaz ao problema.

Exemplo-88:

Dadas 32 pedras de pesos distintos, prove que 35 pesagens num balança de dois pratos sãosuficiente para determinar quais são as duas pedras mais pesadas.

Solução

(LMO - 1989) Inicialmente, dividimos as 32 pedras em 16 pares e, fazendo 16 pesagens, separamoso conjunto das 16 pedras mais pesadas. Agora, dividimos as 16 pedras mais pesadas em parese, com 8 pesagens, determinamos as 8 mais pesadas dentre as 16 mais pesadas. Repetindo oprocesso, dividimos as 8 pedras mais pesadas em pares e, com quatro pesagens, encontramos a4 mais pesadas. Repetindo o processo, podemos determinar a mais pesada das pedras depois de16+8+4+2+1 = 31 pesagens.Durante as pesagens, a segunda pedra com mais peso só poderia ser eliminada pela pedra mais pesada.Nesse momento, você conhece a pedra mais pesada e as 6 mais pesadas. Agora, fique só com as 5mais pesadas. Extraia as 5 pedras que foram pesadas contra a pedra mais pesada durante as pesagens.Nesse momento, é fácil de encontrar a mais pesado dessas 5 pedras em apenas 4 pesagens sucessivas,fazendo a pesagem duas a duas e eliminando a mais leve.

Exemplo-89:

Um polígono convexo possui 2n lados. Prove que o polígono contém no mínimo n diagonaisque não são paralelas a qualquer um de seus lados.

Page 77: NOTAS DE AULA - UFRGS

77

Solução

(SAMO - 1996)Como o polígono possui 2n lados, ele possui 2n.(2n−3)2 = n(2n− 3) diagonais.

Agora, observe que, para cada lado, existem no máximo (n− 2) diagonais paralelas a ele. Destemodo, existem no máximo 2n(n−2) diagonais paralelas a algum lado. Portanto, existem no mínimon(2n−3)−2n(n−2) = n diagonais que não são paralelas a qualquer lado.

Exemplo-90:

Na década de 1960, tinha um programa semanal de televisão muito popular nos EstadosUnidos chamado "Let’s Make a Deal"("Vamos fazer um acordo"). A cada semana, em umdeterminado momento no programa, o apresentador, Monty Hall, apresentava ao competidortrês portas fechadas.

Atrás de uma das portas, havia um prêmio substancial (vamos supor que fosse um carro)e atrás das outras, não havia nada substancial (vamos imaginar que tivesse um bode). Oapresentador, Monty Hall, pedia que o participante escolhesse uma das porta. Claramente, achance de o competidor escolher a porta com o prêmio era de 1 em 3. Para causar impactoaos telespectadores, em vez de simplesmente abrir a porta escolhida para revelar o que estápor trás, o apresentador, Monty Hall, abria uma das duas portas que o competidor não tinhaescolhido, revelando que ela não escondia o prêmio (e sim um bode).

É claro que o apresentador sabia onde o prêmio estava, por isso ele podia fazer isso. Ele entãooferecia ao competidor a oportunidade de mudar sua escolha para a outra porta que, naquelamomento, permanecia fechada.Qual é a estratégia mais lógica, ficar com a porta escolhida inicialmente ou mudar de porta?Com qual das duas portas ainda fechadas o concorrente tem mais probabilidades de ganhar?Por quê?

Solução

Existem três portas: A,B e C.Quando o competidor escolheu uma delas, digamos a porta A, a chance de que ela seja a premiadaé de 1

3 . Como consequência, a probabilidade de que tenha errado, ou em outras palavras, de que oprêmio esteja nas outras duas portas B ou C é de 2

3 .Pode-se comprovar isso somando a probabilidade de cada uma das outras portas ou simplesmentesabendo que a probabilidade de que haja um prêmio é sempre 1.O importante é ter em mente que a chance de o prêmio estar nas outras portas que ele não escolheu éde 2

3 .Entendendo isso, basta ver que o apresentador abrirá sem erro uma dessas outras duas portas quecontém um bode, digamos que seja a porta B. Ao fazer isso, ele está dando uma informação valiosa:se o prêmio estava nas outras portas que não escolheu (B ou C), então agora ele só pode estar naporta que o competidor não escolheu e não foi aberta, ou seja, a porta C. Ou seja, se o competidorerrou ao escolher uma porta - e as chances disto são de 2/3 - então ao abrir uma das outras portasnão-premiadas o apresentador está lhe dizendo onde está o prêmio. Toda vez que o competidor tiverescolhido inicialmente uma porta errada, ao trocar de porta irá com certeza ganhar. Como as chancesde que tenha errado em sua escolha inicial são de 2

3 , se trocar suas chances de ganhar serão de 23 - e

Page 78: NOTAS DE AULA - UFRGS

78 Capítulo 3. Problemas Diversos

por conseguinte a chance de que ganhe se não trocar de porta é de apenas 13 . É assim mais vantajoso

trocar de porta.ObservaçãoSuponha que, em vez de 3 houvessem 1000 portas, numeradas de 1 a 1000, e o competidorescolheu a porta de número 1000.

Todos as portas contém um bode, exceto uma, aquela que tem o carro.

Quando o competidor escolhe uma porta, em seguida, Monty Hall abre todas as outras portas, excetouma... e dá a oportunidade para o competidor mudar a escolha para a outra porta. Se fosse você,mudaria? É possível um pensamento do tipo "Escolhi a porta correta no meu primeiro palpite."Mas,qual é a probabilidade de que isso fosse verdade? Resposta: 1

1000 ... Há uma chance de 999 de que ocarro não está atrás da porta que o competidor escolheu. E se o carro não está atrás da porta que ocompetidor escolheu, ele deve estar por trás da última porta que Monty deixou sem abrir. Em outraspalavras, Monty ajudou, deixando uma porta para que o competidor mude, que tem uma chance de999 de ter o carro por trás dela. Então nesse caso, se você mudar, você teria uma chance de 999

1000 deganhar o carro. Portanto, é melhor mudar.

Exemplo-91:

Dois jogadores, A e B, disputam um jogo em que jogam alternadamente. Uma jogada de A éescolher um ponto do plano, que não esteja pintado, e pintá-lo de vermelho. Uma jogada de Bé escolher 10 pontos do plano, que não estejam pintados, e pintá-los de verde. O jogador Avence se existem três pontos vermelhos que sejam vértices de um triângulo equilátero.O jogador B pode impedir do jogador A de vencer a partida?

Solução

A resposta é não.O primeiro jogador, A, pode sempre vencer, não importa como o jogador B jogue. Para isso, em suasn primeiras jogadas ele escolhe pintar de vermelho pontos sobre uma mesma reta. Agora, observe quepara cada par destes n pontos existem dois pontos, um de cada lada da reta, que o segundo jogador,B, tem de pintar de verde, para evitar que o jogador A vença.O números de tais pontos é igual a 2×

(n2

). Por outro lado, o jogador B pode pintar no máximo 10n

pontos de verde. Agora, 2×(n

2

)> 10n, que é o mesmo que n(n−1)> 10n. Ou seja, n−1 > 10.

Portanto, o primeiro jogador vence no seu 12o movimento.

Exemplo-92:

Ana e Célia disputam o jogo seguinte, em que jogam alternadamente. Ana deve pintar, oude azul ou de vermelho, cada ponto que esteja sobre um dado círculo. Célia deve escolhertrês dos pontos pintados de modo que eles determinem um triângulo cujas medidas de seusângulos internos sejam 30,50 e 100 graus, respectivamente. Célia vence se esse triângulopossui os vértices de mesma cor. Ana vence se Célia não consegue fazer a escolha.Quem vence: Ana ou Célia?

Solução

Page 79: NOTAS DE AULA - UFRGS

79

Ana vence.Basta ela marcar 6 pontos sobre o círculo, de modo que este fique dividido em seis arcos de mesmocomprimento. Agora ela pinta alternadamente de azul e vermelho cada um dos arcos, de formaconsecutiva, tendo o cuidado de pintar em cada arco o ponto inicial da cor do arco, mas deixar oponto final com a cor do próximo arco. Quando Célia quiser escolher os vértices de seu triângulo, osextremos do lado oposto ao ângulo de 600, ela deve determinar um arco de 600 sobre o círculo. Pelapintura feita, estes extremos devem necessariamente possuir cores distintas.

Exemplo-93:

Numa caixa, temos 25 fichas iguais, numeradas de 1 até 25. Você pode retirar da caixauma ficha por vez, sem reposição, e pode continuar a retirar fichas até que o produto de doisnúmeros de duas fichas retiradas seja um quadrado perfeito.Qual é o número mínimo de fichas que você deve retirar para ter certeza de obter um par defichas cujo produto é um quadrado perfeito?

Solução

Seja S = {1,2,3, . . . ,24,25}. Partimos o conjunto S em subconjuntos disjuntos com as seguintesduas propriedades:(i) o produto de quaisquer dois elementos de um subconjunto é um quadrado perfeito; (ii) o produto dequaisquer dois elementos escolhidos em subconjuntos distintos não é um quadrado perfeito. Assim,escrevemos:

S = {1,4,9,16,25}∪{2,8,18}∪{3,12}∪{5,20}∪{6,24}∪{7}∪{10}∪

∪{11}∪{13}∪{14}∪{15}∪{17}∪{19}∪{21}∪{22}∪{23}

.Ou seja, partimos o conjunto S em 16 subconjuntos disjuntos. Se escolhermos uma ficha de cadaum dos subconjuntos não teremos obtido um par de fichas satisfazendo ao problema. Mas, com maisuma ficha vamos obter um par de fichas cujo produto é um quadrado perfeito. Portanto, a resposta é16+1 = 17.

Exemplo-94:

Dois jogadores, A e B, disputam um jogo, em que jogam alternadamente. Inicialmente,escreve-se no quadro-negro o número natural 2. Uma jogada consiste em somar ao últimonúmero, n, no quadro-negro um divisor próprio de n. Quem atingir um número maior doque ou igual 2010 vence. O jogador A começa o jogo.Quem vence, A ou B? Qual é a estratégia para vencer?

Solução

O jogador A possui uma estratégia vencedora.A estratégia do jogador A consiste em jogar deixando para seu adversário sempre um número ímpar.Observe que um divisor próprio de um número ímpar é no máximo um terço daquele número, pois omenor divisor deste número é 3. Deste modo, o jogador B só pode somar no máximo 1

3 do últimonúmero escrito no quadro-negro.Toda vez que o jogador B vai jogar, encontra um número ímpar e soma a ele um divisor dele, queé ímpar também, resultando sempre num número par. O jogador A por sua vez soma sempre umdivisor de um número par. Assim, ele pode jogar a metade deste número e prossegue assim até obterpela primeira vez um número maior do que ou igual a 1340. Uma vez obtido esse número x, ojogador A soma a ele sua metade, atingindo então um número maior do que 2010. (Observe que:

Page 80: NOTAS DE AULA - UFRGS

80 Capítulo 3. Problemas Diversos

x+x2 ≥ 2010⇒ x≥ 1340).

Exemplo-95:

Dois jogadores, A e B, disputam o seguinte jogo, em que jogam alternadamente.Escrevem-se no quadro-negro uma sequência de 49 números inteiros consecutivos quaisquer.Uma jogada consiste em apagar um desses números. No final, restam somente dois números:a e b. O jogador A ganha se MDC(a,b) = 1 e B ganha se MDC(a,b)> 1. O jogador Acomeça o jogo.(a) Quem ganha: A ou B? Qual a estratégia para vencer?(b) Se, em vez de 49 números inteiros consecutivos fossem 50, quem ganha: A ou B?Qual a estratégia para vencer?

Solução

(a) O jogador A ganha.Como a quantidade de inteiros é ímpar, o jogador divide os 49 primeiros números em pares de inteirossucessivos, deixando o número final sozinho. Ele inicia apagando o número que ficou sozinho. Se ojogador B apaga qualquer número de um par, então A apaga o outro. O que sobra no final é um par(n,n+1), com MDC(n,n+1) = 1.(b) O jogador B ganha.A estratégia do jogador B é apagar todos os números ímpares, deixando dois números ímparesmúltiplos de 3. Por sua vez, o jogador A sempre apaga um número par. Antes da penúltima jogadarestam quatro números no quadro-negro: dois números pares, p e q, e dois números ímpares, i ej. Se A apagar um número ímpar, então B apaga o outro número ímpar, restando os dois númerospares. Se A apaga um dos números pares, então B apaga o outro número par, restando i e j, comMDC(i, j)≥ 3.

Exemplo-96:

Todo membro de uma seqüência de números, a partir do segundo, é igual a soma do termoprecedente com a soma de seus dígitos. O primeiro número da seqüência é 1.Diga, justificando, se o número 2010 pertence à seqüência.

Solução

A resposta é não.Seja a1,a2,a3,a4, . . . a sequência de números. Observe que:

a1 = 1,

a2 = 2 = 3.0+2,

a3 = 4 = 3.1+1,

a4 = 8 = 3.2+2,

a5 = 16 = 3.5+1,

a6 = 23 = 3.7+2,

a7 = 28 = 3.9+1,

. . . . . . . . . . . . . . .

Para cada inteiro m, seja S(m) a soma dos dígitos de m. Sabe-se que, na divisão por 3, os númerosm e S(m) deixam o mesmo resto. Observando os números da seqüência encontrados acima, temosque os restos da divisão por 3 dos termos da seqüência são alternadamente 1 e 2. De fato, se

Page 81: NOTAS DE AULA - UFRGS

81

an = 3k+ 1, com k um número inteiro, então an+1 = an + S(an) = (3k+ 1)+ (3q+ 1) = 3 j+ 2,onde q e j são números inteiros.Se an = 3k+2, com k um númro inteiro, então an+1 = an +S(an) = (3k+2)+(3q+2) = 3 j+1,onde q e j são números inteiros.Deste modo, de fato o resto da divisão dos termos da sequência por 3 é ou 1 ou 2. Mas, 2010deixa resto zero na divisão por 3. Portanto, 2010 não aparece na sequência.

Exemplo-97:

No aniversário de Maria, José quer dar-lhe um presente e, para isso, propõe o jogo se-guinte, no qual eles jogam alternadamente. Ele escreve no quadro-negro os números inteiros0,1,2,3,4, . . . ,255,256. Maria começa o jogo. Ela escolhe 27 dos números escritos eapaga-os. José escolhe 26 números dentre o que sobraram e apaga-os. Em seguida, Mariaescolhe 25 dentre os números restantes a apaga-os. E eles prosseguem assim até que Joséescolhe 20 = 1 número e apaga-o. Como são apagados

27 +26 +25 +24 + . . .+21 +20 = 28˘1 = 255

números, restam no quadro-negro dois números, a e b. José paga para Maria uma quantia,em reais, correspondente ao valor |a−b|.Se os jogadores fazem sempre suas melhores jogadas, qual é o maior valor que Maria temcerteza que ganha?

Solução

(CRUX) Maria ganha no máximo 24 ou 216 reais.A estratégia de Maria é apagar sempre os números que estão nas posições pares. Com isso, ela aumentaa distância entre eles, aumentando o valor de |a−b|. Depois das jogadas de números 1,2,3 e 4,a distância entre os números vizinhos, que estão ainda escritos no quadro-negro, é 2,4,6,8 e 16,respectivamente, o que garante a Maria receber o máximo possível, independente de como José joga.

Exemplo-98:

Daniel tem uma balança de dois pratos e dez pedras cujos pesos são todos números inteirosentre 1 até 10, inclusive. Ele quer colocar algumas pedras na balança de tal forma que ospratos fiquem equilibrados.(a) Determine qual é o maior e o menor número de pedras que pode colocar. Justifique por queé o maior e mostre um exemplo.(b) Determine a maior quantidade de pedras que se pode colocar se em cada prato se devehaver a mesma quantidade de pedras.

Solução

(RP) (a) É impossível colocar 10 pedras, pois 1+ 2+ 3+ 4+ 5+ 6+ 7+ 8+ 9+ 10 = 55, que éum número ímpar e não pode ser escrito como soma de dois inteiros iguais. Nove pedras podemser usadas, basta retirar uma pedra cujo peso é um número ímpar. Por exemplo, se retirarmos apedra 1, podemos colocar num dos pratos as pedras 8+ 9+ 10 = 27 e no outro prato as pedras2+3+4+5+6+7 = 27.(b) O máximo possível de pedras que podemos usar é 8. Por exemplo, descartamos as pedras 9 e 10e usamos 1+4+5+8 = 18 num dos pratos e 2+3+6+7 = 18 no outro prato.

Page 82: NOTAS DE AULA - UFRGS

82 Capítulo 3. Problemas Diversos

Exemplo-99:

Um motorista planeja uma viagem que vai passar por um trecho de 800 km no deserto,que não possui postos para abastecer. Ele sabe que seu carro só pode armazenar 50 litrosde gasolina e tem um rendimento de 10 km por litro. O motorista pode deixar gasolinaarmazenada em barris, no costado da rodovia, em pontos distintos dessa região desértica. Porexemplo, com o tanque cheio com 50 litros de gasolina ele pode percorrer 100 km, deixararmazenado 30 litros no ponto que chegou e voltar ao ponto de partida para reabastecer otanque de gasolina. O motorista decide realizar a viagem e chega ao primeiro posto antes dodeserto com tanque vazio.Pode o motorista atravessar o deserto se neste primeiro posto de abastecimento antes dodeserto a oferta total de gasolina é de 140 litros? E se fosse 180 litros?

Solução

(OMP) Com a oferta de 140 litros não é possível o motorista atravessar o deserto.Para atravessar o deserto se necessita armazenar gasolina no quilômetro 300 ou mais adiante, casocontrário, se percorreria mais de 500 km sem abastecer, o que é impossível, pois a autonomia do carroé de no máximo 500 km. Chamemos este ponto de depósito de combustível de P. Para armazenargasolina em P o motorista tem de percorrer pelo menos duas vezes a distância do primeiro posto aoponto P, que é pelo menos 2×300 km = 600 km. Como a distância a ser percorrida é de 800 km,vemos que é impossível atravessar o deserto com uma oferta menor do que 140 litros. Por outro lado,não se pode atravessar o deserto com uma oferta exatamente igual a 140 litros, simplesmente porqueé impossível armazenar gasolina em P e voltar com esta quantidade.

Com 180 litros é possível atravessar o deserto seguindo o seguinte roteiro:(a) O motorista faz três viagens para armazenar gasolina no ponto P, situado a 50 km da partida,armazenando 40 litros em cada uma das viagens;(b) Após estas três viagens, abastece o tanque no primeiro posto com 30 litros, volta ao ponto P,chegando com 25 litros;(c) Do ponto P, avança duas vezes a um ponto Q a 100 km dele, armazenando ali 30 litros emcada viagem;(d) De volta ao ponto P abastece com 45 litros que estão armazenados ali e segue até o ponto Qcom 35 litros de gasolina;(e) A partir de Q, chegando num ponto R, armazenando ali 20 litros de gasolina e volta ao pontoQ;(f) Abastece os 45 litros de gasolina restantes em Q, avança até R, chegando com 30 litros,abastece ali com os 20 litros existentes no local e completa a viagem.

Exemplo-100:

Qual é o valor máximo que se pode obter ao dividir um número natural de três dígitos pelasoma de seus dígitos?Justifique porque não se pode obter um valor maior.

Solução

(Olimpíada Matemática Rioplatense) Todo número natural de três dígitos é da forma

a ·100+b ·10+ c, com a, b, c ∈ {0, 1, 2, 3, 4, 5, 6, 7, 8, 9} e a 6= 0.

Vamos provar quea ·100+b ·10+ c

a+b+ c≤ 100.

Page 83: NOTAS DE AULA - UFRGS

83

De fato, basta observar que a · 100+ b · 10+ c ≤ a · 100+ b · 100+ c · 100 = 100(a+ b+ c), o queimplica a·100+b·10+c

a+b+c ≤ 100.Agora, observe que o valor 100 é atingido quando a = 1, b = c = 0. Ou seja, quando o número éigual a 100:

1001+0+0

= 100.

Exemplo-101:

O quadrado da figura a seguir, que está dividido em quadradinhos de dimensão 2×2, foiconstruído com palitos de fósforos de comprimento 2 cm.

Encontrar o tamanho do maior quadrado dividido em quadradinhos 2× 2 que podemosconstruir com 1999 palitos de fósforos. Quantos palitos sobraram? Quantos palitos tem emcada lado deste quadrado?

Solução

(OMA - 1999) Observe que, para construir um quadrado onde num lado usamos n palitos de fósforos,dividido em quadradinhos 2× 2, precisamos utilizar n(n+ 1) palitos de fósforos na horizontal en(n+1) palitos de fósforos na vertical. Isto é, para construir um quadrado como se quer, usamosn(n+1)+n(n+1) = 2n(n+1) palitos de fósforos.O que queremos é encontrar o maior inteiro n para o qual

2n(n+1)≤ 1999.

Basta observar que, como

2 ·31 ·32 = 1984 < 1999 < 2 ·32 ·33 = 2112,

o maior valor possível para n = 31. Neste caso, se usa 1994 palitos de fósforos, sobrando1999− 1984 = 15 palitos. O lado do quadrado que se constrói utilizando-se os 1984 palitospossui 2 ·31 = 62 palitos de fósforos.

Page 84: NOTAS DE AULA - UFRGS
Page 85: NOTAS DE AULA - UFRGS

4. ABREVIATURAS

4.1 Abreviaturas• AMO - Australian Mathematical Olympiad• AUMO - All Union Mathematical (URRS)• BW - Baltic Way Mathematical Contest• CMO - Canadian Mathematical Olympiad• CRUX - Crux Mathematicorum (Revista de resolução de problemas - Canadá)• HMITMT - Havard-MIT Mathematics Tournaments• IMO - International Mathematical Olympiad• KMO - Kiev Math Olympiad• LMO - Lenigrad Mathematical Olympiad• OMA - Olimpiada Matemática Argentina• OMM - Olimpiada Mexicana de Matemática• OMP - Olimpiada de Matemática do Peru• RP - Olimpiada Rioplatense de Matemática• SAMO - South African Mathematics Olympiad• TT - Tournament of the Tows

Page 86: NOTAS DE AULA - UFRGS
Page 87: NOTAS DE AULA - UFRGS

5. Bibliografia

• ANDREESCU, Titu; SAVCHEV, Svetoslav - Mathematical Miniatures. The MathematicalAssociation of America. Anneli Lax New Mathematical Library, Volume #43. Wasington . 2003.

• ANGEL, D - http://math.stackexchange.com/questions/731193/how-to-win-this-game

• KISACANIN, BRANISLAV - Mathematical Problems and Proof - Combinatorial, NumberTheory, and Geometry. Kluwer Academic Publishers. New York. 2002

• BRIGGS, William - Ants, bikes & clocks - Problems solving for undergraduates. Society forIndustrial and Applied Mathematics - Siam. Philadelphia. 2005

• EWING, J. - Paul Halmos: In his own words. Notice of AMS. Vol. 54 . Number 9. October 2007.

• GUZMÁN, O. M. - Aventuras Matemáticas. Gradiva. Lisboa. 1990.

• CRUX MATHEMATICORUM. Canadian Mathematical Society.

• MARTÍN, J. E. - Resolucion de Problemas Matemáticos (Vol 3). Centro de Professores y Recur-sos. Salamanca. 1999. (Em PDF)

• POSAMENTIER, A.S.; WOLFGANG, S - The Art of Problem Solving. Corwin Press. ThousandOaks. 1996.

• POSAMENTIER, A.S. - The Art of Solving Problems. Vienna 2-06 (Em PPT)

• POSAMENTIER, A.S. - Problem Solving in Mathematics - Grade 3-6 Powerful Strategies toDeeper. Corwin Press. Thousand Oaks. 2009.

• ZEITZ, P. - The Art and Craft of Problem Solving. Second Edition. Wiley. Danvers. 2007